OB Midterm Nclex Questions

¡Supera tus tareas y exámenes ahora con Quizwiz!

A pregnant patient scheduled for an amniocentesis asks the nurse how the placenta is not punctured during the procedure. What should the nurse respond to the patient?

"A sonogram to locate it will be done first."

The school nurse is presenting a lecture to adolescents to teach them how conception occurs. Which statement by the nurse would accurately describe this process?

"Conception usually occurs when the ovum is in the outer third of the fallopian tube."

A pregnant woman recently diagnosed with the genital herpes virus asks the nurse for more information on the virus. Which responses by the nurse would be appropriate? Select all that apply.

"Transmission is through contact of infected mucous membranes." "Infections may be transmitted by individuals unaware that they have it." "The virus remains quiet until a stressful event occurs to reactivate it." Genital herpes simplex is transmitted by contact of mucous membranes or breaks in the skin with visible or nonvisible lesions. Most genital herpes infections are transmitted by individuals unaware that they have an infection. Kissing, sexual contact (including oral sex), and vaginal birth are means of transmission. The virus remains latent until stimuli such as fever, stress, ultraviolet radiation, or immunosuppression occurs and reactivates it.

A nurse is preparing a teaching plan for a female client diagnosed with genital ulcers. Which instructions would the nurse include in this teaching plan? Select all that apply.

"Use a condom when having sexual intercourse with any noninfected partner." "Avoid having sex when any ulcers are present." "Air dry any lesions with a hair dryer on the low setting."

During her first prenatal visit, a woman asks why she is being screened for syphilis. Which of the following is the best response by the healthcare provider? Choose 1 answer: 1) "If you have the infection, the safest time to treat you is during the first trimester." 2) "Syphilis can be transferred from you to your baby through the placenta." 3) "We must report all cases of syphilis to the health department." 4) "If you test positive you may need to be admitted to the hospital for care."

2) "Syphilis can be transferred from you to your baby through the placenta." Penicillin, the drug of choice for the treatment of syphilis during pregnancy, has not shown to have adverse effects on the fetus. Syphilis is transmitted vertically from mother to baby during pregnancy and/or delivery, so routine prenatal care includes testing for syphilis.

Which of the following is a presumptive sign or symptom of pregnancy? 1) Restlessness 2) Elevated mood 3) Urinary frequency 4) Low backache

3) Urinary frequency Urinary frequency occurs during early pregnancy secondary to pressure on the bladder by the expanding uterus. This is one of the presumptive signs of pregnancy. Restlessness or elevated mood is not a sign of pregnancy. As hormones increase during pregnancy, the mood might change, but it is not indicative of pregnancy. Low backache is frequently experienced by many women during the third trimester of pregnancy secondary to the change in their center of gravity, but it is not a presumptive sign of pregnancy.

A patient asks the healthcare provider about the benefits of receiving the human papillomavirus (HPV) vaccine. Which statement is the most appropriate response by the healthcare provider? Choose 1 answer: 1) "You will need to have a booster vaccination each year." 2) "You will no longer need to get a routine cervical exam." 3) "The HPV vaccine will protect you from all types of the virus." 4) "The HPV vaccine can help prevent cervical cancer."

4) "The HPV vaccine can help prevent cervical cancer." The vaccine is effective in preventing cervical cancer associated with certain types of HPV, but because it does not protect against all HPV types, regular cervical screenings are recommended.

A woman asks "why do I have to tell my sexual partner about chlamydia? I thought this was a "female" disease." Which of the following is the nurse's best response? 1) "You are right, but notifying your sexual partner is required by law due to the fact this is a reportable disease." 2) "The condition is not transmittable; however, you need to let your partner know what you are going through." 3) "The condition has the potential to cause other infections." 4) "The condition is transmittable to men and may cause infertility."

4) "The condition is transmittable to men and may cause infertility."

A woman in labor is to receive continuous internal electronic fetal monitoring. The nurse reviews the woman's medical record to ensure which of the following as being required? A) Intact membranes B) Cervical dilation of 2 cm or more C) Floating presenting fetal part D) A neonatologist to insert the electrode

B) Cervical dilation of 2 cm or more For continuous internal electronic fetal monitoring, four criteria must be met: ruptured membranes, cervical dilation of at least 2 cm, fetal presenting part low enough to allow placement of the electrode, and a skilled practitioner available to insert the electrode. (pg. 452)

At 37 weeks gestation a patient calls the labor and delivery floor and thinks she is in labor. What statement should the nurse recognize as an assessment finding for true labor? a) Bloody mucus in the toilet once earlier in the day b) Contraction, regular and lasting longer and stronger c) Contractions, irregular, lasting 15 to 20 seconds d) Scant amount of thick white vaginal discharge, no odor

Contraction, regular and lasting longer and stronger True labor contraction will progressively get worse and last longer. The pain will come to a point where the woman will not be able to walk or talk through the contractions. Irregular contractions, bloody show, and white vaginal discharge are normal for pregnancy but do not indicate true labor.

A pregnant woman tends not to eat for long periods of time because of her busy work schedule. What process safeguards her fetus from becoming hypoglycemic during this time?

Somatomammotropin helps to regulate glucose levels

During a clinical rotation at a prenatal clinic, a client asks a nurse what causes certain birth defects. The nurse replies that they can be caused by teratogens. What does the severity of the defects depend on? Select all that apply.

When during development the conceptus is exposed to the teratogen The particular teratogen to which the fetus is exposed

A client with genital herpes asks the nurse about what to expect with the infection. Which response would be most appropriate? a) "Even though you don't have symptoms, you could still spread the infection." b) "You might have to try several different medications before finding one that works." c) "You can expect other outbreaks, each of which will be longer than the first." d) "Once you take the medication, the infection will be gone for good."

a) "even though you don't have symptoms, you could still spread the infection." Genital herpes can be transmitted during asymptomatic periods of viral shedding. Herpes recurs because after the initial infection, the virus remains dormant in the ganglia of the nerves that supply the area. Symptoms usually are more severe with the initial outbreak. Subsequent episodes usually are shorter and less intense. When the virus is active, shedding viral particles are infectious. Herpes infection is a highly contagious STI that is controllable but not curable. Herpes virus responds well to the antiviral drugs acyclovir, valacyclovir, and famciclovir.

The blood volume in pregnant women increases by what percent? a) 40-50 percent b) 30-40 percent c) 20-30 percent d) 10-20 percent

a) 40-50 percent

pregnant client has heard about Down syndrome and wants to know about the risk factors associated with it. Which of the following would the nurse include as a risk factor? a) Advanced maternal age b) Advanced paternal age c) Family history of condition d) Recurrent miscarriages

a) Advanced maternal age

A nurse working in a community health education program is assigned to educate community members about sexually transmitted infections (STIs). Which nursing strategy should be adopted to prevent the spread of STIs in the community? a) Discuss limiting the number of sex partners. b) Promote use of oral contraceptives. c) Emphasize not sharing personal items with others. d) Emphasize the importance of good body hygiene.

a) Discuss limiting the number of sex partners. As a nursing strategy to prevent the spread of STIs, the nurse should discuss reducing the number of sex partners to diminish the risk of acquiring STIs. Oral contraceptives are not effective in preventing STIs, and barrier methods (condoms, diaphragms) should be promoted. The nurse should counsel and encourage sex partners of persons with STIs to seek treatment. Maintaining good body hygiene or not sharing personal items with others does not reduce the risk of spreading STIs.

The hormone responsible for the initiation of lactation is what? a) Progesterone b) Prolactin c) Oxytocin d) Estrogen

b) Prolactin

A client in her 29th week of gestation complains of dizziness and clamminess when assuming a supine position. During the assessment, the nurse observes there is a marked decrease in the client's blood pressure. Which of the following interventions should the nurse implement to help alleviate this client's condition? a) Keep the client's legs slightly elevated. b) Place the client in an orthopneic position. c) Place the client in the left lateral position. d) Keep the head of the client's bed slightly elevated.

c) Place the client in the left lateral position.

A nurse should teach a client who is pregnant for the first time that she should be able to feel the baby move at about how many weeks gestation? a) 14 to 16 b) 10 to 12 c) 12 to 14 d) 18 to 20

d) 18 to 20

Before becoming pregnant, a woman's heart rate averaged 72 beats per minute. The woman is now 15 weeks pregnant. The nurse would expect this woman's heart rate to be approximately: a) 95 beats per minute b) 90 beats per minute c) 100 beats per minute d) 85 beats per minute

d) 85 beats per minute

A nurse urges a pregnant patient at the first prenatal office visit to begin taking iron supplements immediately. What is the rationale for this intervention? a) To maintain proper blood glucose levels b) To prevent megalohemoglobinemia c) To reduce the risk for hypertension d) To avoid anemia

d) To avoid anemia

A client reports genital ulcers and a diagnosis of syphilis. Which nursing interventions should the nurse implement when caring for the client? Select all that apply

• Have the client urinate in water if urination is painful. • Instruct the client to wash her hands with soap and water after touching lesions. • Instruct the client to wear nonconstricting, comfortable clothes. Explanation: The nurse should instruct the client to wear nonconstricting clothes and to wash her hands with soap and water after touching lesions to avoid autoinoculation. If urination is painful because of the ulcers, instruct the client to urinate in water but to avoid extremes of temperature such as ice packs or hot pads to the genital area. The client should abstain from intercourse during the prodromal period and when lesions are present. The ulcer disappears during the latency period.

It is important to test anyone who has had sex with an individual tests positive for gonorrhea, within how many days of the diagnosis? 1) 60 days 2) 90 days 3) 180 days 4) 365 days

1) 60 days All sex partners within 60 days prior to discovery of the infection should be contacted, tested, and treated.

A woman who is 3 months pregnant enjoys a slow, long walk daily. Which of the following would be most appropriate for her concerning this for the remainder of her pregnancy? a) Reduce walking to half a block daily. b) Stop and rest every block. c) Engage in aerobics for greater benefits. d) Continue this as long as she enjoys it.

Continue this as long as she enjoys it. Walking is an excellent exercise during pregnancy because it is low impact and increases venous circulation. Exercise should be maintained as long as it is comfortable, but intensity should not increase over what is normally performed.

A woman complains of constant redness and itching of her palms early in her pregnancy. She fears that she is suffering an allergic reaction and asks the nurse whether this is normal. Which of the following should the nurse mention? a) It is caused by increased estrogen levels and should disappear in time b) It is a sign of fatigue; she should get more rest c) It is a sign of high blood pressure; she should start an anti-hypertensive drug d) It is likely an allergic reaction to a lotion, which should be avoided during pregnancy

It is caused by increased estrogen levels and should disappear in time Palmar erythema, or palmar pruritus, occurs in early pregnancy and is probably caused by increased estrogen levels. Constant redness or itching of the palms can make a woman believe she has developed an allergy. Explain that this type of itching in early pregnancy is normal. She may find lotion to be soothing. As soon as a woman's body adjusts to the increased level of estrogen, the erythema and pruritus disappear. This condition is not a sign of fatigue or high blood pressure.

During a previous prenatal visit, the nurse focused on the importance of adequate nutritional intake with a pregnant patient. Which assessment findings indicate that this teaching has been effective? (Select all that apply.)

Normal muscle reflexes Shiny hair Smooth tongue

A woman has heard that hypotension can be a problem during pregnancy, but she is not sure what it is or what causes it. The nurse explains that it is simply a temporary bout of low blood pressure due to impaired blood return to the heart. It is commonly caused by sleeping in a position that causes compression of the vena cava blood vessel. To avoid this condition, which of the following should the nurse mention? a) Sleep flat on your back b) Sleep face down c) Sleep on your side d) Sleep with your feet elevated

Sleep on your side Supine hypotension is a symptom that occurs when a woman lies on her back and the uterus presses on the vena cava, impairing blood return to her heart. A woman experiences an irregular heart rate and a feeling of apprehension. To relieve the problem is simple: if a woman turns or is turned onto her side, pressure is removed from the vena cava, blood flow is restored, and the symptoms quickly fade. To prevent the syndrome, advise pregnant women to always rest or sleep on their side, not their back. Sleeping face down is not advised, and sleeping with the feet elevated would not prevent compression of the vena cava.

A woman is concerned that orgasm will be harmful during pregnancy. Which of the following statements is most factual? a) Most women do not experience orgasm during pregnancy. b) Venous congestion in the pelvis makes orgasm painful. c) Orgasm during pregnancy is potentially harmful. d) Some women experience orgasm intensely during pregnancy.

Some women experience orgasm intensely during pregnancy. Because of pelvic congestion, orgasm may be achieved more readily by pregnant women than nonpregnant women.

The nurse is providing prenatal care to a young couple that is pregnant with their first child. In what period of development would the nurse explain to the couple that most congenital defects would occur? a) All periods are equally vulnerable b) The period of the embryo c) The period of the fetus d) The period of the zygote

The period of the embryo During the period of the embryo, which lasts until the eighth week after conception, the embryo is in what is called the critical phase of human development. During these weeks, all the organs and structures of the human are formed and are most susceptible to damage.

The fetal structure with the highest oxygen concentration is which? a) Pulmonary vein b) Umbilical artery c) Umbilical vein d) Ductus arteriosus

Umbilical vein The umbilical vein carries oxygenated blood from the placenta to the fetus; the umbilical artery carries de-oxygenated blood from the fetus to the placenta. The ductus arteriosus shunts blood from the right side to the left in order to bypass the deflated lungs. The pulmonary vein drains the deflated lungs.

Which of the following is a presumptive sign or symptom of pregnancy? a. Restlessness b. Elevated mood c. Urinary frequency d. Low backache

Urinary frequency

A pregnant client complains to the nurse of shortness of breath when sleeping. The nurse informs the client that this is normal and occurs because the growing fetus puts pressure on the diaphragm. Which measure should the nurse suggest to help alleviate this problem? a) Use extra pillows b) Avoid spicy food c) Avoid overeating d) Lie on a firmer mattress

Use extra pillows The nurse should instruct the client to use extra pillows at night to keep her more upright. The nurse can instruct the client to use a firmer mattress if the client is experiencing backache. The nurse can ask the client to avoid overeating and ingesting spicy food in case the client is experiencing heartburn.

A pregnant client is scheduled to undergo chorionic villi sampling (CVS) to rule out any birth defects. Ideally, when should this testing be completed? a) 4-5 weeks of gestation b) 5-6 weeks of gestation c) 7-9 weeks of gestation d) 10-12 weeks of gestation

10 to 12 weeks of gestation Chorionic villus sampling (CVS) is typically performed between 10 to 12 weeks' gestation. Sometimes it may be offered up to 14 weeks. The test is not conducted before 10 weeks' gestation.

The nurse should administer Rhogam (Rh immune globulin) to the pregnant woman who is Rho(D)-, after which of the following tests? a) NST (Non Stress Test) b) Biophysical Profile c) CST (Contraction Stress Test) d) Amniocentesis

Amniocentesis Amniocentesis is an invasive procedure whereby a needle inserted into amniotic sac to obtain a small amount of fluid. This places the pregnancy at risk for a woman with RhD-negative blood and she should receive RhoGam after the procedure. The CST,NST, and a biophysical profile are non-invasive tests.

A pregnant patient is undergoing a fetal biophysical profile. Which parameter of the profile helps measure long-term adequacy of the placental function? a) Fetal reactivity b) Fetal heart rate c) Fetal breathing record d) Amniotic fluid volume

Amniotic fluid volume A biophysical profile combines five parameters (fetal reactivity, fetal breathing movements, fetal body movement, fetal tone, and amniotic fluid volume) into one assessment. The fetal heart and breathing record measures short-term central nervous system function; the amniotic fluid volume helps measure long-term adequacy of placental function.

A woman in her first trimester is having trouble maintaining adequate nutrition because of nausea and vomiting. She also complains that her heartburn gets worse after eating so she avoids food even when she feels hungry. To help with her nutritional deficit, she is taking a multivitamin supplement. Which substance do you caution her to avoid within 1 hour of ingesting her multivitamin supplement? a) An antacid b) Acetaminophen c) Fatty or fried foods d) Coffee or other caffeinated beverages

An antacid Antacids interfere with the uptake of the vitamin contents. She needs to be encouraged to eat small frequent meals and notify the provider if she is losing weight. Caffeine should be avoided due to increases in blood pressure and diuretic effects. Acetaminophen should be taken only when the provider has approved it. Fatty foods are not healthy, and may make the morning sickness worse.

After teaching a group of students about the different methods for contraception, the instructor determines that the teaching was successful when the students identify which of the following as a mechanical barrier method? (Select all that apply.) A) Condom B) Cervical cap C) Cervical sponge D) Diaphragm E) Vaginal ring

Ans: A, B, C, D Barrier methods include the condom, cervical cap, cervical sponge and diaphragm. The vaginal ring is considered a hormonal method of contraception.

The nursing instructor is preparing a class presentation covering the various hormones and their functions during pregnancy. The instructor determines the class is successful when the class correctly matches which function with hCG? 1) maintains nutrient-rich decidua 2) sustains life of placenta 3) continues progesterone production by corpus luteum 4) provides rich blood supply to decidua

continues progesterone production by corpus luteum The corpus luteum is responsible for producing progesterone until this function is assumed by the placenta. hCG is a fail-safe mechanism to prolong the life of the corpus luteum and ensure progesterone production. Estrogen is responsible for providing a rich blood supply to the decidua. Progesterone helps maintain a nutrient-rich decidua.

A woman at 15 weeks' gestation asks the nurse what the fetus looks like. Which response by the nurse would be most accurate? a) Fingernails and toenails are present. b) The fetus is covered with a white, greasy film called vernix. c) Rhythmic breathing movements are occurring. d) The fetus is about 15 inches in length.

Fingernails and toenails are present. Vernix caseosa, a white, greasy film, covers the fetus at weeks 17 through 20. The fetus reaches a length of approximately 15 inches by weeks 25 to 28. Fingernails and toenails are present between weeks 13 through 15. Rhythmic breathing movements occur between weeks 29 through 32.

Fetal circulation differs from the circulatory path of the newborn infant. In utero the fetus has a hole connecting the right and left atria of the heart. This allows oxygenated blood to quickly pass to the major organs of the body. What is this hole called?

Foramen ovale

A client has been confirmed to be pregnant. She gives a history of two previous full-term normal pregnancies. How will the nurse classify the client's pregnancy history? a) G3, P0 b) G2, P1 c) G3, P2 d) G2, P3

G3, P2 Gravida (G) is the total number of pregnancies the client has had, including the present one, and para (P) is the number of babies born at 20 or more weeks of gestation. Since she gives a history of two previous normal deliveries, she is P2 and not P0, P1, or P3. Because she has had a total of three pregnancies including the present one, she is G3 and not G2.

A nurse is classifying the pregnancy history of a woman who has had five pregnancies: three full-term, one preterm, and one abortion. How would the nurse document this information on the patient chart? a) G5 P1135 b) G4 P3115 c) G5 P3114 d) G5 P3115

G5 P3114 G = gravida or the total number of pregnancies, which in this case equals five. P = para is the outcome of the pregnancies in the following order: full term, preterm, abortions, and living as of today. In this case, P3114.

Yvonne, a 27-year-old client, is in the first trimester of an unplanned pregnancy. She acknowledges that it would be best if she were to quit smoking now that she is pregnant, but states that it would be too difficult given her 13 pack-year history and circle of friends who also smoke. She asks the nurse, "Why exactly is it so important for me to quit? I know lots of smokers who have happy, healthy babies." What can the nurse tell Yvonne about the potential effects of smoking in pregnancy? a) "Smoking during pregnancy places your baby at an increased risk of mental retardation." b) "Smoking during pregnancy means that your child will be born with a dependence on nicotine and will have to endure a period of withdrawal in his or her first days of life." c) "Babies of women who smoke tend to weigh significantly less than other infants." d) "Smoking is unhealthy for anyone's heart, but your baby faces an especially high risk of heart trouble if you smoke while you're pregnant."

"Babies of women who smoke tend to weigh significantly less than other infants." Smoking during pregnancy is linked with low birth weight but not cardiac anomalies, mental retardation, or nicotine dependence.

A woman opts to use a diaphragm for contraception. Which instruction would be most important for the nurse to provide? "Keep the diaphragm in place for no more than 4 hours after intercourse." "Replace the diaphragm every 6 months." "Wet the diaphragm with water first before inserting it." "Have your diaphragm refitted if you lose 10 pounds (4.5 kilograms) or more."

"Have your diaphragm refitted if you lose 10 pounds (4.5 kilograms) or more." Diaphragms should be refitted after pregnancy, abdominal or pelvic surgery, or weight loss or gain of 10 pounds (4.5 kilograms) or more. A diaphragm usually is replaced every 1 to 2 years. A diaphragm should remain in place for at least 6 hours after intercourse. A contraceptive sponge, not a diaphragm, should be wetted with water before insertion.

A client who is 15 weeks' gestation is attending prenatal classes and asks the nurse, "What changes in development has my baby made?" Which statements would the nurse include in the response? Select all that apply. 1)"Your baby's lungs are ready for life outside the uterus." 2) "Soft hair covers your baby's head." 3) "Your baby makes sucking motions now with its mouth." 4) "You will be able to detect quickening." 5) "The increase in weight of your baby has stopped now."

"Soft hair covers your baby's head." "Your baby makes sucking motions now with its mouth." "You will be able to detect quickening." During weeks 13 through 16, a fine hair called lanugo develops on the head. The fetus makes active movement, sucking motions are made with the mouth, weight quadruples, and fetal movement (also known as quickening) is detected by mother. Alveoli of the lungs have not developed, and thus lungs are not ready for life outside the womb.

A client has been recently diagnosed with genital herpes and asks, "Why am I having so many recurring outbreaks of the infection?" What is the nurse's best response? a) "Your body is responding to the bacterial infection." b) "The antibiotic medication requires adjusting." c) "This is the expected course of the infection." d) "Stress-reducing strategies may help prevent the outbreaks."

"Stress-reducing strategies may help prevent the outbreaks." Recurrent genital herpes outbreaks are triggered by precipitating factors such as emotional stress and stress-reducing exercises may help. No cure exists, but antiviral drug therapy not antibiotics helps to reduce or suppress symptoms, shedding, and recurrent episodes. Genital herpes is a recurrent, lifelong "viral" not bacterial infection.

The nursing instructor asks a nursing student to list the characteristics of the amniotic fluid. The student responds correctly by listing which as characteristics of amniotic fluid? Select all that apply. 1) Allows for fetal movement 2) Surrounds, cushions, and protects the fetus 3) Maintains the body temperature of the fetus 4) Can be used to measure fetal kidney function 5) Prevents large particles such as bacteria from passing to the fetus 6) Provides an exchange of nutrients and water products between the mother and the fetus.

1) Allows for fetal movement 2) Surrounds, cushions, and protects the fetus 3) Maintains the body temperature of the fetus 4) Can be used to measure fetal kidney function The amniotic fluid surrounds, cushions, and protects the fetus. It allows the fetus to move freely and maintains the body temperature of the fetus. In addition, the amniotic fluid contains urine from the fetus and can be used to assess fetal kidney function. The placenta prevents large particles such as bacteria from passing to the fetus and provides as exchange of nutrients ans waste products between the mother and the fetus.

During a routine antepartal visit, a pregnant woman reports a white, thick vaginal discharge. What would the nurse do next? 1) Ask the woman if she is having any itching or irritation. 2) Advise the woman about the need to culture the discharge. 3) Tell the woman that this is entirely normal. 4) Check the discharge for evidence of ruptured membranes.

1) Ask the woman if she is having any itching or irritation. Although vaginal secretions increase during pregnancy, the nurse would need to ascertain if this discharge is the normal leukorrhea of pregnancy or if it is a monilial vaginitis, which is common during pregnancy. The nurse needs additional information to conclude that the woman's report is normal. A culture may or may not be necessary. There is no evidence to suggest that her membranes have ruptured.

Which of the following is not true about breast-feeding? 1) Breast-fed infants experience more obesity and allergies 2) Breast milk is perfectly suited to the infant's nutritional needs 3) Breast milk contains maternal antibodies to stimulate infant's immunity 4) Breast-feeding enhances maternal bonding and attachment

1) Breast-fed infants experience more obesity and allergies Based on research, breast-feeding prevents childhood and adult obesity and infants who are breast-fed experience fewer allergies versus those that are bottle-fed. Responses "2," "3," and "4" are all true statement about breast-feeding. These responses represent the advantages of breast-feeding.

The healthcare provider is teaching a youth group about how to avoid acquiring a sexually transmitted disease (STD) if they are sexually active. Which of the following is the best method of prevention? Choose 1 answer: 1) Consistent condom use 2) Human papillomavirus (HPV) vaccination 3) Use of spermicidal creams 4) Douching after intercourse

1) Consistent condom use None of these methods is 100% effective, but one method provides the best protection. The HPV vaccination is effective in preventing some strains of the human papillomavirus. Although not 100% effective, routine condom use is the best way to prevent acquiring a sexually transmitted disease (STD).

A woman whose pregnancy has been confirmed asks the healthcare provider when her baby is due. The first day of her last menstrual period was March 20. Using Naegele's rule, the healthcare provider estimates the date of birth (EDB) to be which of the following? Choose 1 answer: 1) December 27 2) January 17 3) December 20 4) February 13

1) December 27 To estimate the EDB, begin with the first day of the woman's last menstrual period and add 7 days. Next subtract 3 months. The EDB for this woman is December 27. Add a year to the year of the last menstrual period to find correct year of the EDB.

Which of the following is the major goal of genetic counseling? 1) Diagnose and determine the role of heredity 2) Reinforce previously presented test data 3) Emphasize good communication skills 4) Offer referral to community support groups

1) Diagnose and determine the role of heredity The correct response is "1" since the goals of genetic counseling include diagnosis, determining the role of genetics, assessing risks and options, and opening up lines of communication with the couple affected. The other responses (2, 3, and 4) are included in genetic counseling, but are not the major goals of it.

When evaluating the fetal heart rate (FHR) pattern for a woman in active labor, the healthcare provider notes periodic decelerations in the FHR with the following characteristics: Gradual decrease in FHR Onset just prior to the uterine contraction 120 beats/minute at the peak of the uterine contraction Waveform inversely mirrors uterine contraction What is the healthcare provider's priority action? Choose 1 answer: 1) Document this as a normal finding 2) Assist the woman to lay on her left side 3) Administer oxygen by nasal cannula 4) Assess the degree of cervical dilation

1) Document this as a normal finding The slowing deceleration is caused by compression of the fetal head and subsequent vagal response. When the onset of the deceleration occurs early in relation to the uterine contraction and mirrors the contraction, the waveform is an early deceleration. Early decelerations are not clinically significant, so the healthcare provider will continue routine monitoring.

The cardinal movements of labor include which of the following? Select all that apply. 1) Extension and rotation 2) Descent and engagement 3) Presentation and position 4) Attitude and lie 5) Flexion and expulsion

1) Extension and rotation 2) Descent and engagement 5) Flexion and expulsion The cardinal movements of labor by the fetus include engagement, descent, flexion, international rotation, extension, external rotation, and expulsion only. The other choices describe the various fetal positions.

The nurse is describing the pre-embryonic stage of fetal development to a group of students. Place the events in their proper sequence for this stage. 1) Formation of the zygote 2) Formation of the morula 3) Cleavage 4) Formation of blastocyst 5) Implantation

1) Formation of the zygote 3) Cleavage 2) Formation of the morula 4) Formation of blastocyst 5) Implantation The pre-embryonic stage begins with fertilization and the formation of a zygote. Mitosis or cleavage occurs. After four cleavages, the 16 cells appear as a solid ball of cells or morula. With additional cell division, the morula divides into specialized cells, the blastocyst and trophoblast. The trophoblast attaches itself to the surface of the endometrium and implantation occurs

A woman is admitted to the medical unit with a report of sudden, severe, sharp, pain in the lower abdomen and an absence of menses of 8 weeks. When interviewing the patient, about which of the following will the healthcare provider inquire? Choose 1 answer: 1) History of chlamydial infection 2) Frequency of sexual activity 3) Sexual orientation 4) Intravenous drug use

1) History of chlamydial infection The healthcare provider will want to inquire about risk factors that may be associated with the patient's presentation. The patient is showing signs of a complication from pelvic inflammatory disease (PID). Chlamydia trachomatis can ascend into the upper genital tract, and cause PID. This patient is experiencing an ectopic pregnancy secondary to PID.

When obtaining a blood test for pregnancy, which hormone would the nurse expect the test to measure? 1) Human chorionic gonadotropin (hCG) 2) Human placental lactogen (hPL) 3) Follicle-stimulating hormone (FSH) 4) Luteinizing hormone (LH)

1) Human chorionic gonadotropin (hCG) hCG is produced by the trophoblast (outermost layer of the developing blastocyst) and maintains the ovarian corpus luteum (remainder of ovarian follicle after ovulation) by keeping levels of progesterone and estrogen elevated until the placenta can take over that function. hCG is secreted early after conception to signal that fertilization has taken place. Without fertilization, hCG is not detected. Thus, it is the basis for pregnancy tests. hPL is the hormone secreted by the placenta to prepare the breasts for lactation. It is also an antagonist to insulin, competing for receptor sites that force insulin secretion to increase to meet the body's demands. FSH is secreted by the anterior pituitary gland to stimulate the ovary to mature an ovum for ovulation. It is not detected during pregnancy tests. LH is secreted by the pituitary gland. An increase in LH occurs immediately before ovulation and is responsible for release of the ovum. It is not the basis for pregnancy tests.

A nurse is providing care to a pregnant woman. To promote optimal outcomes, the nurse would engage in which activity? Select all that apply. 1) Individualized assessment 2) Assistance with social coordination 3) Counseling 4) Authoritarian decision making 5) Teaching

1) Individualized assessment 3) Counseling 5) Teaching Nurses contribute to the success of prenatal care through individualized assessment, counseling, and educating. Assistance with social coordination and authoritarian decision making are not associated with successful prenatal care.

The healthcare provider is caring for a patient diagnosed with gonorrhea. Which of the following assessment findings could indicate the patient is experiencing a complication from the disease? Choose all answers that apply: 1) Joint pain and stiffness 2) Vaginal pruritus and dyspareunia 3) Excessive bruising 4) Dysuria, urgency, or urinary frequency 5) Pelvic or abdominal pain

1) Joint pain and stiffness 2) Vaginal pruritus and dyspareunia 4) Dysuria, urgency, or urinary frequency 5) Pelvic or abdominal pain Infection with Neisseria gonorrhoeae can lead to a variety of clinical symptoms. The infection can cause problems in the urogenital tract or may become disseminated to other parts of the body. Dysuria, urgency, or urinary frequency are common symptoms of urethritis. Vaginal pruritus and dyspareunia (painful intercourse) is associated with upper genital tract infections. Pelvic or abdominal pain is associated with pelvic inflammatory disease, a complication gonorrhea infection. Joint pain and stiffness is associated with disseminated gonorrhea.

Which of the following would be considered risk factors for psychological well-being in pregnancy? Select all that apply: 1) Limited support system and network of friends and family 2) Introverted personality at any point in the pregnancy 3) Ambivalence any time during the pregnancy 4) High levels of stress due to family discord 5) History of previous high-risk pregnancy with complications 6) Depression prior to pregnancy and on medication

1) Limited support system and network of friends and family 4) High levels of stress due to family discord 5) History of previous high-risk pregnancy with complications 6) Depression prior to pregnancy and on medication All of these factors will influence the pregnancy outcome in a negative way and the ability to cope with them might become challenging. These factors would be "red flags" in a client's history and counseling and education might be needed during this pregnancy. Responses "2" and "3" are both normal emotions that are universally experienced by most pregnant women. They would not indicate a problem.

When teaching a group of students about chlamydia, which of the following points is most important for the healthcare provider to emphasize? Choose 1 answer: 1) Most people infected with chlamydia are unaware that they are infected 2) Burning and pain with urination is a frequent symptom of chlamydial infections 3) Chlamydia is the least common of all the major sexually transmitted diseases 4) Good handwashing technique is the best way to prevent chlamydial infections

1) Most people infected with chlamydia are unaware that they are infected Chlamydia is known as a 'silent' infection because most infected people are asymptomatic and lack abnormal physical examination findings.

Interventions that are underutilized in promoting a normal birth. Select all that apply. 1) Oral nutrition and fluids in labor 2) Open glottis pushing in the second stage of labor 3) Skin-to-skin contact after birth for infant bonding 4) Routine artificial rupture of membranes (amniotomy) 5) Labor induction with Pitocin given intravenously 6) Routine episiotomy to shorten labor length

1) Oral nutrition and fluids in labor 2) Open glottis pushing in the second stage of labor 3) Skin-to-skin contact after birth for infant bonding All of these are evidence-based interventions that are physiologically sound without placing the mother or the neonate in any danger. Food and clear fluids provide hydration and nutrition and give comfort to laboring women. Fasting during labor will increase gastric acid production. Open glottis while pushing allows the woman's body to sense the urge to push naturally. Skin-to-skin contact promotes mother-infant bonding and warmth. Incorrect responses would include "4," "5," and "6" since these are artificial means to speed up the labor process which places the mother and newborn in jeopardy. Amniotomy may be associated with umbilical cord prolapse and fetal heart rate decelerations. Episiotomy is associated with an increase in third- and fourth-degree perineal lacerations, discomfort, and healing delays. Induction with Pitocin may cause tetanic contractions causing hypoxia to the fetus.

The healthcare provider is caring for a newborn diagnosed with a gonococcal infection. Which of the following will the healthcare provider expect to observe? Choose all answers that apply: 1) Purulent conjunctivitis 2) Swollen eyelids 3) Hard and fixed lymph nodes 4) Generalized maculopapular rash 5) Corneal ulcerations

1) Purulent conjunctivitis 2) Swollen eyelids 5) Corneal ulcerations If the mother has gonorrhea, the newborn can acquire a gonococcal infection after rupture of the membranes or during delivery. The bacteria will cause localized infection of mucosal surfaces. The eye is the most frequent site of gonococcal infection in the newborn, the major cause of ophthalmia neonatorum (newborn conjunctivitis). Infection typically causes a purulent conjunctivitis, with profuse exudate and swelling of the eyelids. The infection can extend to the cornea, leading to ulceration, scarring, and visual impairment or even blindness.

During pregnancy, which of the following should the expectant mother reduce or avoid? 1) Raw meat or uncooked shellfish 2)Fresh, washed fruits and vegetables 3) Whole grains and cereals 4) Protein and iron from meat sources

1) Raw meat or uncooked shellfish Consuming raw meat can increase the pregnant woman's risk of picking up toxoplasmosis, a parasitic infection that can be passed on to her fetus. Although toxoplasmosis may go unnoticed in the pregnant woman, it may cause abortion or result in the birth of an infant with the disease. Uncooked shellfish may contain high levels of mercury, which can damage the fetal central nervous system. Some raw or undercooked can also be contaminated with Listeria, which may result in abortion, stillbirth, or severe illness of the newborn. Raw or undercooked food items should be avoided during pregnancy.

Which of the following combination contraceptives has been approved for extended continuous use? 1) Seasonale 2) Triphasil 3) Ortho Evra 4) Mirena

1) Seasonale Seasonale is one of several FDA-approved long-term oral contraceptives that is packaged to provide 84 days of continuous protection. Although any oral contraceptive can be taken continuously, the FDA has not approved this, and it would be considered an "off-label" use. "2" is incorrect: this product has not gained FDA approval for continuous use; it is to be left in 3 weeks and then removed for 1 week to create monthly cycles. "3' is incorrect response: the FDA has not given approval to use this transdermal patch on a continuous basis; it is placed on the skin for 3 weeks and removed for 1 week. "4" is incorrect: this implantable device is protective for 5 years, but it is not a combination contraceptive; it releases synthetic progesterone only, not estrogen.

Select the most reliable test to diagnose syphilis. 1) Serology 2) Oral swab 3) Urine culture 4) Chest x-ray

1) Serology Although PCR tests have been developed for syphilis, serology tests remain the mainstay for diagnosis. Two general types of serology tests are available: nonspecific (nontreponemal) tests and the specific treponemal tests. The other tests are not reliable.

When teaching about HIV transmission, which of the following does the nurse explain that the virus cannot be transmitted by? 1) Shaking hands 2) Sharing drug needles 3) Sexual intercourse 4) Breastfeeding

1) Shaking hands The HIV virus is not spread through casual contact between individuals. HIV is spread through unprotected sexual intercourse, breast-feeding, blood contact, or shared needles or sex toys. 2 is incorrect: HIV can be spread by sharing injection equipment because the user can come into contact with HIV-positive blood. 3 is incorrect: sexual intercourse (unprotected vaginal, anal, or oral) poses the highest risk of HIV transmission. 4 is incorrect: the newborn can receive the HIV virus through infected breast milk. HIV-positive women are advised not to breast-feed to protect their offspring from getting an HIV infection.

Which of the following assessment findings helps the nurse confirm that the patient has genital warts? 1) Soft, raised lesions on external genitalia 2) Dark red, hard lesions on external genitalia 3) Dark brown or black papules on external genitalia 4) Hard white lesions with dark centers

1) Soft, raised lesions on external genitalia Genital warts typically present as soft, raised, fleshy lesions on the external genitalia, including the penis, scrotum, perineum, and perianal skin.

When determining the frequency of contractions, the nurse would measure which of the following? 1) Start of one contraction to the start of the next contraction 2) Beginning of one contraction to the end of the same contraction 3) Peak of one contraction to the peak of the next contraction 4) End of one contraction to the beginning of the next contraction

1) Start of one contraction to the start of the next contraction Frequency is measured from the start of one contraction to the start of the next contraction. The duration of a contraction is measured from the beginning of one contraction to the end of that same contraction. The intensity of two contractions is measured by comparing the peak of one contraction with the peak of the next contraction. The resting interval is measured from the end of one contraction to the beginning of the next contraction.

A female client presents to the clinic with a rash on the palms of her hands, symptoms of hair loss, and the development of elevated, red-brown lesions that have begun to ulcerate and produce a foul discharge. Based on these symptoms, the nurse plans to test the client for which of the following sexually transmitted disease? 1) Syphilis 2) Chlamydia 3) Genital herpes 4) Gonorrhea

1) Syphilis The timing of the second stage of syphilis varies even more than that of the first, lasting from 1 week to 6 months. The symptoms of a rash (especially on the palms, mucous membranes, meninges, lymph nodes, stomach, soles, and liver), fever, sore throat, stomatitis, nausea, loss of appetite, and inflamed eyes may come and go for a year but usually last for 3 to 6 months. Secondary manifestations may include some loss of hair and condylomata lata. These lesions are elevated, red-brown lesions that may ulcerate and produce a foul discharge. They are 2 to 3 cm in diameter, contain many spirochetes, and are highly infectious.

A patient is admitted with a diagnosis of tertiary syphilis. When planning care for this patient, which of the following should the healthcare provider keep in mind? Choose all answers that apply: 1) The patient's heart should be auscultated for murmurs 2) Contact precautions should be initiated immediately 3) The patient is now in the latent stage of the disease 4) The patient should be observed for changes in personality 5) Nuchal rigidity is sometimes present in tertiary syphilis 6) Hearing loss or tinnitus may be reported by the patient

1) The patient's heart should be auscultated for murmurs 4) The patient should be observed for changes in personality 5) Nuchal rigidity is sometimes present in tertiary syphilis 6) Hearing loss or tinnitus may be reported by the patient Tertiary syphilis may cause cardiovascular problems such as aortic valve regurgitation. Hearing loss, with or without tinnitus, may be present as part of the neurologic symptoms of neurosyphilis. Patients with symptomatic syphilitic meningitis may report headache, confusion, nausea and vomiting, and stiff neck (nuchal rigidity). In the early stage of tertiary syphilis, general paresis is associated with symptoms of forgetfulness and personality change.

A pregnant client at 26 weeks' gestation has arrived for a routine prenatal visit. Which assessments should the nurse prioritize? Select all that apply. 1) blood pressure 2) weight 3) edema of the face and hands 4) urine testing 5) blood glucose level

1) blood pressure 2) weight 4) urine testing 5) blood glucose level Up to the 28th week of gestation, follow-up visits involve assessment of the client's blood pressure and weight, urine testing for protein and glucose, along with fundal height and fetal heart rate. Between weeks 24 and 28, a blood glucose level is obtained. Assessment for edema is typically done between 29 and 36 weeks' gestation; however, edema of the face and hands should be reported if noted sooner.

A woman just gave birth to a healthy term newborn. Upon assessing the umbilical cord, the nurse would identify which structures as normal? Select all that apply. 1) one vein 2) one ligament 3) two ligaments 4) two veins 4) two arteries 5) one artery

1) one vein 4) two arteries

A primigravida who is at 38 weeks gestation calls the clinic stating she felt several contractions in her abdomen during the past hour. She reports that the contractions were irregular and they decreased after she took a walk around the block. Based on this information, what is the healthcare provider's best response? Choose 1 answer: 1) "Come to the hospital when your contractions are regular and about 1 minute apart." 2) "Come to the hospital when your contractions are regular and about 5 minutes apart." 3) "You should wait 1 to 2 hours and then come to the hospital." 4) "Birth is imminent so you should come to the hospital immediately."

2) "Come to the hospital when your contractions are regular and about 5 minutes apart." True labor produces contractions that cause the cervix to begin to dilate (open up) and efface (get thinner). In order make changes in the cervix, contractions need to gradually increase in frequency. Contractions of true labor are regular and gradually increase in frequency, duration, and intensity. They usually start in the back and move to the front of the abdomen, and they are not relieved by walking. The beginning (latent) stage of labor is characterized by contractions every 10 to 30 minutes, progressing to every 5 to 7 minutes. False labor contractions are irregular, are usually felt only in the front of the abdomen, and often stop or decrease with activity.

The healthcare provider is teaching a patient who is diagnosed with genital herpes about the disease. Which of the following will be included in the teaching plan for this patient? Choose all answers that apply: 1) "You will not be contagious when you are taking your antiviral medications." 2) "This infection also increases your risk of human immunodeficiency (HIV) virus infection." 3) "There is no cure for genital herpes but outbreaks can be shortened with medication." 4) "Your infection is caused by a corkscrew bacteria called a spirochete." 5) "Transmission of the virus can occur even if there are no visible herpes sores." 6) "Use condoms between outbreaks to reduce the risk of transmission."

2) "This infection also increases your risk of human immunodeficiency (HIV) virus infection." 3) "There is no cure for genital herpes but outbreaks can be shortened with medication." 5) "Transmission of the virus can occur even if there are no visible herpes sores." 6) "Use condoms between outbreaks to reduce the risk of transmission." The virus, herpes simplex virus type 1 (HSV-1) or type 2 (HSV-2), can become active without causing symptoms like blisters or sores. Condoms should always be used, although they cannot totally eliminate the likelihood of transmission. Antiviral suppressive therapy reduces but does not completely eliminate viral shedding. Genital ulcerative disease caused by the herpes virus makes it easier to transmit and acquire HIV infection.There is an estimated 2- to 4-fold increased risk of acquiring HIV if exposed to HIV when genital herpes is present. There is no cure for herpes. Antiviral medications can, however, prevent or shorten outbreaks during the period of time the person takes the medication. And daily use of antiviral medications can reduce the likelihood of transmission to partners.

The nurse should include which statement to a pregnant client found to have a gynecoid pelvis? 1) Your type of pelvis has a narrow pubic arch." 2) "Your type of of pelvis is the most favorable for labor and birth." 3) "Your type of pelvis is a wide pelvis, but has a short diameter." 4) "You will need a cesarean section because this type of pelvis is not favorable for a vaginal delivery."

2) "Your type of of pelvis is the most favorable for labor and birth." A gynecoid pelvis is a normal female pelvis and is the most favorable for successful labor and birth. An android pelvis (resembling a male pelvis) would be unfavorable for labor because of the narrow pelvic planes. An anthropoid pelvis has an outlet that is adequate , with a normal or moderately narrow pubic arch. A platypelloid pelvis (flat pelvis) has a wide transverse diameter, but the anteroposterior diameter is short, making the outlet inadequate.

A laboring woman is admitted to the labor and birth suite at 6 cm dilation. She would be in which phase of the first stage of labor? 1) Latent 2) Active 3) Transition 4) Early

2) Active Cervical dilation of 6 cm indicates that the woman is in the active phase of the first stage of labor. In this phase, the cervix dilates from 3 to 7 cm with 40% to 80% effacement occurring. During the latent phase, the cervix dilates from 0 to 3 cm. During the transition phase, the cervix dilates from 8 to 10 cm. The first stage of labor is divided into three phases: latent, active, and transition. There is no early phase.

On the second day postpartum after a vaginal delivery, a woman develops a number of lesions on her labia that are vesicular and painful. The woman is diagnosed with genital herpes. Which of the following will the healthcare provider include when caring for the newborn? Choose all answers that apply: 1) Assist with serial chest radiographs 2) Administer intravenous antiviral medications 3) Apply antibiotic ointment to both eyes 4) Culture the conjunctivae, nasopharynx, and rectum 5) Institute contact precautions 6) Isolate the baby from other infants

2) Administer intravenous antiviral medications 4) Culture the conjunctivae, nasopharynx, and rectum 5) Institute contact precautions 6) Isolate the baby from other infants Genital herpes is caused by the herpes simplex virus (HSV). Mucous membranes are exposed to the virus during a vaginal birth. Contact precautions also should be used for infants who are hospitalized with HSV infection, and the baby should be isolated from other infants on the unit.

The nurse is preparing her teaching plan for a woman who has just had her pregnancy confirmed. Which of the following should be included in it? Select all that apply. 1) Prevent constipation by taking a daily laxative 2) Balance your dietary intake by increasing your calories by 300 daily 3) Continue your daily walking routine just as you did before this pregnancy 4) Tetanus, measles, mumps, and rubella vaccines will be given to you now 5) Avoid tub baths now that you are pregnant to prevent vaginal infections 6) Sexual activity is permitted as long as your membranes are intact 7) Increase your consumption of milk to meet your iron needs

2) Balance your dietary intake by increasing your calories by 300 daily 3) Continue your daily walking routine just as you did before this pregnancy 5) Avoid tub baths now that you are pregnant to prevent vaginal infections 6) Sexual activity is permitted as long as your membranes are intact All of these are sound recommendations for a healthy pregnancy. Pregnant women are to avoid taking medications so the nurse would not recommend a daily laxative. Increasing fluids, exercise and fiber in the diet are better choices to prevent constipation. The vaccinations shown are contraindicated for pregnant women by the CDC. Adverse outcomes could result if given while pregnant. Milk does not provide a good source of iron. Typically, iron and folic acid are supplemented in prenatal vitamins and good food sources to prevent anemia include meats, cereals, cooked beans, spinach, broccoli, and wheat germ.

A woman with HPV is likely to present with which nursing assessment finding? 1) Profuse, pus-filled vaginal discharge 2) Clusters of genital warts 3) Single painless ulcer 4) Multiple vesicles on genitalia

2) Clusters of genital warts The human papillomavirus (HPV) causes warts in the genital region. HPV is a slow-growing DNA virus belonging to the papilloma group. Types 6 and 11 usually cause visible genital warts. Other HPV types in the genital region (16, 18, 31, 33, and 35) are associated with vaginal, anal, and cervical dysplasia. 1 is incorrect: a pus-filled discharge is not typical of an HPV infection, but rather a chlamydial or gonococcal STI. 3 is incorrect: a single painless ulcer would be indicative of primary syphilis rather than an HPV infection. 4 is incorrect: multiple vesicles would indicate a herpes outbreak, not an HPV infection. The woman would also experience tingling, itching, and pain in the affected area.

A patient has been diagnosed with primary syphilis (Stage I). When assessing the patient, which of these findings will the healthcare provider anticipate? Choose 1 answer: 1) Reddish rash on the palms of the hands 2) Firm and painless genital ulcers 3) Sore throat and swollen lymph glands 4) Muscle weakness and visual changes

2) Firm and painless genital ulcers Although there may be multiple sores, the first indication someone is in the primary stage of syphilis is the appearance of a single chancre, which is a round, firm, and painless sore found at the location where syphilis entered the body.

A pregnant client close to term comes into the clinic for an exam. The woman complains about experiencing shortness of breath. The nurse knows that this complaint can be explained as the: 1) Fetus is needing more oxygen now that his/her size is larger. 2) Fundus of the uterus is high and pushing the diaphragm upwards. 3) Woman is experiencing an allergic reaction because of high histamine levels. 4) Oxygen partial pressure concentration is lower in the third trimester.

2) Fundus of the uterus is high and pushing the diaphragm upwards. The growing gravid uterus displaces the diaphragm upward which forces the entire thoracic cavity to compensate by increasing its dimensions so that more air can be inspired. Shortness of breath develops in most women during the last month of pregnancy. Response "A" is incorrect since the placenta is essentially the "lungs" for the growing fetus and thus the exchange of oxygen and carbon dioxide takes place there, not via the maternal respiratory system. Response "C" is incorrect since there isn't a histamine response in a normal pregnancy to cause an allergy. Response "D" is incorrect since the partial pressure of oxygen increases throughout pregnancy, not lessens.

The healthcare provider is assessing a pregnant woman during her first prenatal visit. The patient reports she has a 5-year-old at home who was delivered at 39 weeks gestation. Her last pregnancy ended at 12 weeks gestation due to a spontaneous abortion. She delivered a set of twins at 22 weeks gestation. The twins died within 12 hours of birth. How will the healthcare provider document this patient's obstetrical history using the GTPAL system? Choose 1 answer: 1) G4 T1 P1 A1 L1 2) G4 T1 P2 A1 L1 3) G5 T2 P1 A0 L3 4) G3 T2 P2 A0 L2

2) G4 T1 P2 A1 L1 "G" stands for gravida: the number of times a woman is pregnant, regardless of duration. "T" stands for the number of babies born at term (after 37 weeks gestation). "P" stands for the number of babies born preterm (after 20 weeks but before 37 weeks gestation). Multiple births count as one pregnancy. "A" stands for the number of pregnancies that terminate in either a spontaneous abortion (miscarriage) or therapeutic abortion. "L" stands for the number of living children.

A client comes to the clinic for pregnancy testing. The nurse explains that the test detects the presence of which hormone? 1) Human placental lactogen (hPL) 2) Human chorionic gonadotropin (hCG) 3) Follicle stimulating hormone (FSH) 4) Thyroid stimulating hormone (TSH)

2) Human chorionic gonadotropin (hCG)

Which fetal lie is most conducive to a spontaneous vaginal birth? 1) Transverse 2) Longitudinal 3) Perpendicular 4) Oblique

2) Longitudinal A longitudinal lie places the fetus in a vertical position, which would be the most conducive for a spontaneous vaginal birth. A transverse lie does not allow for a vaginal birth because the fetus is lying perpendicular to the maternal spine. A perpendicular lie describes the transverse lie, which would not be conducive for a spontaneous birth. An oblique lie would not allow for a spontaneous vaginal birth because the fetus would not fit through the maternal pelvis in this side-lying position.

A couple reports that their condom broke while they were having sexual intercourse last night. What would you advise to prevent pregnancy? 1) Inject a spermicidal agent into the woman's vagina immediately. 2) Obtain emergency contraceptives and take them immediately. 3) Douche with a solution of vinegar and hot water tonight. 4) Take a strong laxative now and again at bedtime.

2) Obtain emergency contraceptives and take them immediately. If EC is taken within 72 hours after unprotected sexual intercourse, pregnancy will be prevented by inhibiting implantation. The next morning would still afford time to take EC and not become pregnant. 1 is incorrect: it would be too late to use a spermicidal agent to prevent pregnancy, since the sperm have already traveled up into the female reproductive tract. 3 is incorrect: douching with vinegar and hot water 24 hours after unprotected sexual intercourse will not change the course of events; by then it is too late to prevent a pregnancy, and this combination would not be effective anyway. 4 is incorrect: a laxative will stimulate the gastrointestinal tract to produce defecation but will not disturb the reproductive tract, where fertilization takes place.

As the nurse is explaining the difference between true versus false labor to her childbirth class, she states that the major difference between them is: 1) Discomfort level is greater with false labor. 2) Progressive cervical changes occur in true labor. 3) There is a feeling of nausea with false labor. 4) There is more fetal movement with true labor.

2) Progressive cervical changes occur in true labor. Progressive cervical changes occur in true labor. This is not the case with false labor.

Which of the following activities will increase a woman's risk of cardiovascular disease if she is taking oral contraceptives? 1) Eating a high-fiber diet 2) Smoking cigarettes 3) Taking daily multivitamins 4) Drinking alcohol

2) Smoking cigarettes Smoking cigarettes causes vasoconstriction of the blood vessels, increasing peripheral vascular resistance and thus elevating blood pressure. These vascular changes increase the chances of CVD by placing additional pressure on the heart to pump blood with increasing vessel resistance. 1 is incorrect since fiber would be a positive diet addition and assist with elimination patterns and prevent straining, which stresses the heart. 3 is an incorrect response because vitamins do not cause narrowing of the vessel lumen, which places an additional burden on the heart. 4 is an incorrect response since alcohol produces vasodilation and reduces blood pressure. Alcohol in moderation is said to be good for the heart.

What factors would change during a pregnancy if the hormone progesterone were reduced or withdrawn? 1) The woman's gums would become red and swollen and would bleed easily. 2) The uterus would contract more and peristalsis would increase. 3) Morning sickness would increase and would be prolonged. 4) The secretion of prolactin by the pituitary gland would be inhibited.

2) The uterus would contract more and peristalsis would increase. Progesterone is an essential hormone to maintain the pregnancy and prevent early labor. Progesterone decreases systemic vascular resistance early in pregnancy, leading to a decline in blood pressure. It causes relaxation of the uterus and gastrointestinal smooth muscle, resulting in delayed gastric emptying and calming of the uterus. This relaxation mechanism is vital to reduce uterine contractions.

The nurse is conducting a prenatal class for a group of first-time parents in the first trimester. The nurse should point out that the mother should feel the baby move by the end of which week of gestation?

20 weeks

The nurse is measuring the fundal height of a woman who is at 28 weeks' gestation. Which measurement would the nurse expect? a) 12 cm b) 32 cm c) 28 cm d) 18 cm

28 cm Fundal height should be approximately equal to the number of weeks' gestation. In this case, it would be 28 cm.

The nurse is teaching the pregnant woman about nutrition for herself and her baby. Which statement by the woman indicates that the teaching was effective? 1) "Because I am pregnant, I can eat anything I want and not worry about weight gain." 2) "I can eat any seafood that I like because it contains phosphorus, which is a nutrient that pregnant women need." 3) "I will need to take iron supplementation throughout my pregnancy even if I am not anemic." 4) "Milk production requires higher levels of calcium; therefore, if I am going to breastfeed, I must take a calcium supplement during pregnancy."

3) "I will need to take iron supplementation throughout my pregnancy even if I am not anemic." Iron is recommended for all pregnant women because it is almost impossible for the pregnant woman to get what is required from diet alone, especially after 20 weeks' gestation when the requirements of the fetus increase. Pregnant women can get many nutrients from seafood including phosphorus, but there are specific recommendations about types of fish to avoid because of the risk of mercury poisoning. Milk production actually requires higher levels of zinc, which can be obtained from a healthy diet. Calcium requirements do not increase above prepregnancy levels during pregnancy because calcium absorption is enhanced during pregnancy. It can be unsafe for the pregnant woman to eat anything she wants and gain too much weight. A woman who gains too much weight during pregnancy is at risk for delivering a macrosomic baby.

A young woman has contracted the human papillomavirus (HPV) and has developed genital warts. The client is eager to receive an HPV vaccine, citing the large number of television advertisements she has seen. How should the nurse best respond? 1) "That treatment is something you'll have to discuss with your doctor." 2) "Your doctor will likely recommend watchful waiting for the next few weeks." 3) "Unfortunately, these vaccines may be less effective once you have HPV." 4) "These will only be effective if you already received an injection in your teens."

3) "Unfortunately, these vaccines may be less effective once you have HPV." Currently, however, there is no treatment to eradicate the HPV once a person has become infected. Vaccines are ineffective against active infection of a certain type of HPV or colonization of the same. This is true regardless of whether a vaccine was administered in the client's teens. Deferring the client to the physician without providing any anticipatory guidance would be inappropriate.

To confirm a finding of primary syphilis, the nurse would observe which of the following on the external genitalia? 1) A highly variable skin rash 2) A yellow-green vaginal discharge 3) A nontender, indurated ulcer 4) A localized gumma formation

3) A nontender, indurated ulcer The classic chancre in primary syphilis can be described as a painless, indurated ulcer-like lesion at the site of exposure. 1 is incorrect: a highly variable rash is characteristic of secondary syphilis, not primary. 2 is incorrect: this is more descriptive of a trichomoniasis vaginal infection rather than primary syphilis, which manifests with a chancre on the external genitalia. 3 is incorrect: a localized gumma formation on the mucous membranes, such as the lips or nose, is characteristic of late syphilis, along with neurosyphilis and cardiovascular syphilis.

There is no known cure for genital herpes, and methods of treatment are often symptomatic. Pharmacologic management of genital herpes includes which drugs? 1) AZT 2) Topical corticosteroid compounds 3) Acyclovir 4) Nonsteroidal anti-inflammatory drugs

3) Acyclovir The antiviral drugs acyclovir, valacyclovir, and famciclovir have become the cornerstone for management of genital herpes. The other drugs are not used in the treatment of genital herpes.

Which assessment would indicate that a woman is in true labor? 1) Membranes are ruptured and fluid is clear. 2) Presenting part is engaged and not floating. 3) Cervix is 4 cm dilated, 90% effaced. 4) Contractions last 30 seconds, every 5 to 10 minutes.

3) Cervix is 4 cm dilated, 90% effaced. True labor is characterized by contractions occurring at regular intervals that increase in frequency, duration, and intensity. These contractions bring about progressive cervical dilation and effacement. Thus, a cervix dilated to 4 cm and 90% effaced indicates true labor. Rupture of membranes may occur before the onset of labor, at the onset of labor, or at any time during labor and thus is not indicative of true labor. Engagement occurs when the presenting part reaches 0 station; it typically occurs 2 weeks before term in primigravidas and several weeks before the onset of labor or at the beginning of labor for multiparas. Contractions of true labor typically last 30 to 60 seconds and occur approximately every 4 to 6 minutes.

A nursing student is assigned to care for a client in labor. The nursing instructor asks the student to describe fetal circulation, specifically the ductus venosus. Which statement is correct regarding the ductus venosus? 1) Connects the pulmonary artery to the aorta 2) Is an opening between the right and left atria 3) Connects the umbilical vein to the inferior vena cava 4) Connects the umbilical artery to the inferior vena cava

3) Connects the umbilical vein to the inferior vena cava The ductus venosus connects the umbilical vein to the inferior vena cava. The foramen ovale is a temporary opening between the right and left atria. The ductus arteriosus joins the aorta and the pulmonary artery.

After teaching a group of students about fertilization, the instructor determines that the teaching was successful when the group identifies which as the usual site of fertilization? 1) Fundus of the uterus 2) Endometrium of the uterus 3) Distal portion of fallopian tube 4) Follicular tissue of the ovary

3) Distal portion of fallopian tube scientists have determined that conception/fertilization occurs in the upper portion of the fallopian tube. A is an incorrect response because this is where implantation takes place after fertilization has occurred. B is an incorrect response because this describes the inner lining of the uterus, where implantation takes place; not where fertilization of the ovum and sperm occurs. D is an incorrect response because the sperm does not travel outside the fallopian tube to the ovary, but rather meets the ovum for purposes of fertilization in the fallopian tube.

A adolescent asks the nurse if there is anything that would greatly increase their risk of acquiring herpes simplex virus (HSV). Which response would be the most accurate for the nurse to reply? 1) Cigarette smoking 2) Sharing pizza when a HSV-infected friend hands you a slice 3) Engaging in oral-genital sex 4) Hugging a friend who is infected

3) Engaging in oral-genital sex Transmission of HSV is usually contact with infectious lesions or secretions. PSV can also be transmitted though nonsexual routes, including mother to newborn (vertical transmission) and fomites (objects such as clothing, towels, or utensils that harbor the shedding agent).

A male patient arrives at the urgent care clinic reporting the following: painful and swollen testicles, burning with urination, and a yellow-green discharge from the penis. The healthcare provider understands that these symptoms are likely due to which of these problems? Choose 1 answer: 1) Stage 2 syphilis 2) Epididymitis 3) Gonorrhea 4) Trichomoniasis

3) Gonorrhea Men infected with Neisseria gonorrhoeae typically experience sudden onset of painful urination accompanied by a purulent penile discharge that is often yellow-green in color. Testicles may be swollen and tender.

A nurse is working in a women's health clinic. Which woman would genetic counseling be most appropriate for? 1) Had her first miscarriage at 10 weeks 2) Is 30 years old and planning to conceive 3) Has a history with a close relative with Down syndrome 4) Is 18 weeks pregnant with a normal triple screen result

3) Has a history with a close relative with Down syndrome The family history plays a critical role in identifying genetic disorders. A history of a previous child, parents, or close relative with an inherited disease, congenital abnormalities, metabolic disorders, developmental disorders, or chromosomal abnormalities can indicate an increased risk of genetic disorders, therefore referral to genetic counseling is appropriate.

The nurse teaches the pregnant client how to perform Kegel exercises as a way to accomplish which of the following? 1) Prevent perineal lacerations 2) Stimulate labor contractions 3) Increase pelvic muscle tone 4) Lose pregnancy weight quickly

3) Increase pelvic muscle tone Kegel exercises help to tighten and strengthen pelvic floor muscles to improve tone. They can help prevent stress incontinence in women after childbirth. These exercises don't strengthen the perineal area on the outside to prevent lacerations, but rather the internal pelvic floor muscles. Kegel exercises have nothing to do with the start of labor for postdate infants. A drop in progesterone levels and an increase in prostaglandins, not exercise, augment labor. Kegel exercises don't burn calories.

A patient presents to the clinic with a report of joint pain, malaise, fever, and a rash on the palms of the hands. Based on these findings, which action should the healthcare provider implement next? Choose 1 answer: 1) Inquire about the patient's recent sexual activity 2) Draw blood for laboratory analysis 3) Inquire about the development of genital lesions 4) Administer a fever-reducing medication

3) Inquire about the development of genital lesions The patient presentation is consistent with secondary syphilis, so a report of genital lesions would be an indication the patient has had primary syphilis.

Which of the following contraceptive methods offers protection against sexually transmitted infections (STIs)? 1) Oral contraceptives 2) Withdrawal 3) Latex condom 4) Intrauterine system

3) Latex condom It creates a mechanical barrier so that bacteria and viruses cannot gain access to the internal reproductive tract and proliferate. 1 is incorrect: there is no barrier or protection offered by taking an oral pill. Oral contraceptives offer protection against pregnancy by preventing ovulation, but none against STIs. 2 is incorrect: since an infected partner can still transmit the infection through pre-ejaculate fluids, which may contain an active STI. 4 is incorrect: an IUD offers no barrier to prevent entrance of bacteria or viruses into the internal reproductive tract. Because it is an internal device, the string emerging from the external uterine os can actually enhance STI infiltration into the uterus in susceptible women.

Physiologic preparation for labor would be demonstrated by: 1) Decrease in Braxton Hicks contractions felt by mother 2) Weight gain and increase in appetite by mother 3) Lightening, whereby the fetus drops into true pelvis 4) Fetal heart rate accelerations and increased movements

3) Lightening, whereby the fetus drops into true pelvis As labor nears, the fetus gets into position by descending into the maternal true pelvis in preparation for birth. The woman will experience heaviness in her lower pelvis and urinary frequency when this occurs. Response "1" is incorrect since there is an increase in uterine contractions as the uterus becomes more irritable and readies for true labor. Response "2" is incorrect since most women experience a weight loss and a decrease in appetite close to the start of their labor. Response "4" is incorrect due to the fact that the fetus is in a cramped environment at term and has limited room to move around. The fetal heart rate would remain within the normal range of 110 to 160 bpm unless there is a problem.

Which of the following biophysical profile findings indicate poor oxygenation to the fetus? 1) Two pockets of amniotic fluid 2) Well-flexed arms and legs 3) Nonreactive fetal heart rate 4) Fetal breathing movements noted

3) Nonreactive fetal heart rate A nonreactive fetal heart rate is one of the biophysical profile findings that indicate poor oxygenation to the fetus.

What is the first step in determining a couple's risk for a genetic disorder? 1) Observing the client and family over time 2) Conducting extensive psychological testing 3) Obtaining a thorough family health history 4) Completing an extensive exclusionary list

3) Obtaining a thorough family health history uncovering an individual's family history can identify previous genetic disorders that have a high risk for recurrence in subsequent generations. 1 is an incorrect response to this question because observing a patient and their family would be costly and unproductive to diagnose a genetic disorder. This observation would have to take place over several generations to yield results. 2 is an incorrect response because psychological testing might not uncover genetic predispositions to disorders. 4 is an incorrect response because excluding the numerous genetic conditions would be a time-consuming and tedious task.

Which of the following is an example of an autosomal dominant disorder? 1) Phenylketonuria 2) Tay-Sachs disease 3) Polycystic kidney disease 4) Cystic fibrosis

3) Polycystic kidney disease polycystic kidney disease is an example of an autosomal dominant disease because the gene carrying the mutation is located on one of the autosomes of the affected parent and the affected person has a 50% chance of passing the abnormal gene to each of the children. Responses "1," "2," and "4" are all autosomal recessive inheritance disorders that need both parents must carry the abnormal gene and their offspring has a 25% probability of inheriting the two mutant genes.

A woman presents to the clinic because she missed her last menstrual period and thinks she may be pregnant. She reports fatigue, breast tenderness, urinary frequency, and nausea and vomiting in the morning. The healthcare provider will interpret these findings as which of the following changes of pregnancy? Choose 1 answer: 1) Positive 2) Probable 3) Presumptive 4) Possible

3) Presumptive Probable changes of pregnancy are objective signs observed by the healthcare provider. They can be caused by other conditions besides pregnancy. Positive changes of pregnancy include detection of a fetal heartbeat or visualization of a fetus by ultrasound. Presumptive changes of pregnancy are symptoms that are experienced by the woman. These can be caused by other conditions so they are not considered proof of pregnancy.

Reva Rubin identified four major tasks that the pregnant woman undertakes to form a mutually gratifying relationship with her infant. What is "binding in"? 1) Ensuring safe passage through pregnancy, labor, and birth 2) Seeking acceptance of this infant by others 3) Seeking acceptance of self as mother to the infant 4) Learning to give of oneself on behalf of the infant

3) Seeking acceptance of self as mother to the infant Seeking acceptance of self as mother to the infant is the basis for establishing a mutually gratifying relationship between mother and infant. This "binding in" is a process that changes throughout the pregnancy, starting with the mother's acceptance of the pregnancy and then the infant as a separate entity. Ensuring safe passage through pregnancy, labor, and birth focuses on the mother initially and her concern for herself. As the pregnancy progresses, the fetus is recognized and concern for its safety becomes a priority. The mother-infant relationship is not the mother's concern yet. Seeking acceptance of this infant by others includes the world around the mother and how they will integrate this new infant into their world. The infant-maternal relationship is not the focus in this task. Learning to give of oneself on behalf of one's infant focuses on delaying maternal gratification, focusing on the infant's needs before the mother's needs.

The shortest but most intense phase of labor is the: 1) Latent phase 2) Active phase 3) Transition phase 4) Placental expulsion phase

3) Transition phase The transition phase of the first stage of labor occurs when the contractions are 1 to 2 minutes apart and the final dilation is taking place. The transition phase is the most difficult and, fortunately, the shortest phase for the woman, lasting approximately 1 hour in the first birth and perhaps 15 to 30 minutes in successive births. Many women are not able to cope well with the intensity of this short period, become restless, and request pain medications. During the latent phase, contractions are mild. The woman is in early labor and able to cope with the infrequent contractions. This phase can last hours. The active phase involves moderate contractions that allow for a brief rest period in between, helping the woman to be able to cope with the next contraction. This phase can last hours. The placental expulsion phase occurs during the third stage of labor. After separation of the placenta from the uterine wall, continued uterine contractions cause the placenta to be expelled. Although this phase can last 5 to 30 minutes, the contraction intensity is less than that of the transition phase.

The nursing student is preparing to teach a prenatal class about fetal circulation. Which statement should be included in the teaching plan? 1) "One artery carries oxygenated blood from the placenta to the fetus." 2) "Two arteries carry oxygenated blood from the placenta to the fetus." 3) Two arteries carry deoxygenated blood and waste products away from the fetus to the placenta." 4) "Two veins carry blood that is high in carbon dioxide and other waste products away from the fetus to the placenta."

3) Two arteries carry deoxygenated blood and waste products away from the fetus to the placenta." Blood pumped by the embryo's heart leaves the embryo through two umbilical arteries. WHen oxygenated, the blood is returned by one umbilical vein. Arteries carry deoxygenated blood and waste products from the fetus, and the umbilical vein carries oxygenated blood and provides nutrients to the fetus.

Which of the following observations would suggest that placental separation is occurring? 1) Uterus stops contracting altogether. 2) Umbilical cord pulsations stop. 3) Uterine shape changes to globular. 4) Maternal blood pressure drops.

3) Uterine shape changes to globular. After the placenta separates from the uterine wall, the shape of the uterus changes from discoid to globular. The uterus continues to contract throughout the placental separation process and the umbilical cord continues to pulsate for several minutes after placental separation occurs. Maternal blood pressure is not affected by placental separation because the maternal blood volume has increased dramatically during pregnancy to compensate for blood loss during birth.

A woman is in her early second trimester of pregnancy. The nurse would instruct the woman to return for a follow-up visit every: a) 3 weeks b) 2 weeks c) 1 week d) 4 weeks

4 weeks The recommended follow-up visit schedule is every 4 weeks up to 28 weeks, every 2 weeks from 29 to 36 weeks, and then every week from 37 weeks to birth.

A patient diagnosed with trichomoniasis is being treated with metronidazole (Flagyl). When teaching the patient about this medication, which of the following will the healthcare provider include? Choose 1 answer: 1) "Call our office if you experience any tendon pain or tenderness." 2) "Report the occurrence of pain in your upper abdomen immediately." 3) "You should avoid milk or dairy products during therapy." 4) "Do not drink alcohol while you are taking this medication."

4) "Do not drink alcohol while you are taking this medication." Dairy products and some antacids can interfere with tetracycline absorption. Ethanol use while taking metronidazole can cause a disulfiram-like reaction, which is characterized by nausea, vomiting, abdominal cramps, headaches, and flushing.

What comment by a woman would indicate that a diaphragm is not the best contraceptive device for her? 1) "My husband says it is my job to keep from getting pregnant." 2) "I have a hard time remembering to take my vitamins daily." 3) "Hormones cause cancer and I don't want to take them." 4) "I am not comfortable touching myself down there."

4) "I am not comfortable touching myself down there." This vaginal barrier contraceptive device is a dome-shaped rubber cup with a flexible rim that needs to be inserted into the woman's vagina before sexual intercourse. The dome of the diaphragm covers the cervix and the spermicidal cream or jelly applied to the rim prevents sperm from entering the cervix. Women who use this method of contraception must be able to insert the device in their vaginas before each sex act for it to be effective. If the woman is uncomfortable "touching" herself, this is not going to be a successful method and another method should be utilized.

A nurse is teaching personal hygiene care techniques to a client with genital herpes. Which statement by the client indicates the teaching has been effective? 1) "I should rub rather than scratch in response to itching." 2) "I will apply a water-based lubricant to my lesions." 3) "I can pour hydrogen peroxide and water over my lesions." 4) "I will wear loose cotton underwear."

4) "I will wear loose cotton underwear" Wearing loose cotton underwear promotes drying and helps avoid irritation of the lesions. The use of lubricants is contraindicated because they can prolong healing time and increase the risk of secondary infection. Lesions should not be rubbed or scratched because of the risk of tissue damage and additional infection. Cool, wet compresses can be used to soothe the itch. The use of hydrogen peroxide and water on lesions is not recommended.

A male client is diagnosed with gonococcal urethritis. He tells the nurse he had recent sexual contact with a woman but states she did not appear to have any disease. Which of the following is the nurse's best response to the client? 1) "A gonorrhea infection in women only affects the internal organs such as the ovaries; therefore you would not have seen any signs of an active infection." 2) "Women do not develop gonorrhea infections; but they can become carriers and spread the disease to males." 3) "You may have acquired this infection from another route; it can also be spread in droplet form from coughing or sneezing." 4) "Women may not know they have gonorrhea because they may not have symptoms."

4) "Women may not know they have gonorrhea because they may not have symptoms." People with gonorrhea may be asymptomatic and may unwittingly spread the disease to their sexual partners. Many women with gonorrhea are asymptomatic or have minor symptoms that are overlooked. The disease may affect both the genitals and the other reproductive organs and cause complications such as pelvic inflammatory disease (PID). Women who can transmit the disease have active infections.

The nurse is counseling a couple, one of whom is affected by an autosomal dominant disorder. They express concerns about the risk of transmitting the disorder. What is the best response by the nurse regarding the risk that their baby may have for the disease? 1) "You have a one in four (25%) chance." 2) "The risk is 12.5%, or a one in eight chance." 3) "The chance is 100%." 4) "Your risk is 50%, or a one in two chance."

4) "Your risk is 50%, or a one in two chance." Autosomal dominant inheritance occurs when a single gene in the heterozygous state is capable of producing the phenotype. The affected person generally has an affected parent and an affected person generally has a 50% chance of passing the abnormal gene to each of his or her children.

A feeling expressed by most women upon learning they are pregnant is: 1) Acceptance 2) Depression 3) Jealousy 4) Ambivalence

4) Ambivalence The feeling of ambivalence is experienced by most women when they question their ability to become a mother. Feelings fluctuate between happiness about the pregnancy and anxiety and fear about the prospect of new responsibilities and a new family member. Acceptance usually develops during the second trimester after fetal movement is felt by the mother and the infant becomes real to her. Depression is not a universal feeling experienced by most women unless there is a past history of underlying depression experienced by the woman. Jealousy is not a universal feeling of pregnant women. It can occur in partners, because attention is being diverted from them to the pregnancy and the newborn.

A primigravida in preterm labor at 34 weeks gestation feels a sudden gush of water and tells the healthcare provider, "I think my water broke." What is the priority action by the healthcare provider? Choose 1 answer: 1) Prepare for imminent delivery 2) Take the woman's temperature 3) Assess the degree of cervical dilation 4) Assess the fetal heart rate

4) Assess the fetal heart rate The woman's statement is an indication that the amniotic membranes have ruptured. Once the membranes have ruptured there is an increased risk of infection. The woman's temperature will be monitored but this is not the priority. Cervical dilation may be assessed intermittently but this is not the priority. If the fetus is not engaged in the pelvic inlet, there is a danger that the umbilical cord will prolapse. The healthcare provider should assess the fetal heart rate to ensure fetal well-being.

During a clinic visit, a pregnant client at 30 weeks' gestation tells the nurse, "I've had some mild cramps that are pretty irregular. What does this mean?" The cramps are probably: 1) The beginning of labor in the very early stages 2) An ominous finding indicating that the client is about to have a miscarriage 3) Related to over hydration of the woman 4) Braxton Hicks contractions, which occur throughout pregnancy

4) Braxton Hicks contractions, which occur throughout pregnancy The uterus is constantly contracting throughout pregnancy, but the contractions are irregular and not usually felt by the woman, nor do they cause dilation of the cervix. Braxton Hicks contractions are not the start of early labor, since there aren't any measurable cervical changes. They are normal throughout the pregnancy, not an ominous sign of an impending abortion. A woman's hydration status is not related to Braxton Hicks contractions; they occur regardless of her fluid status.

A sexually active 19-year-old presents to the clinic with postcoital bleeding, dysuria, and a yellow discharge. Her cervix upon exam is red and friable. What might the nurse suspect? 1) Cervical cancer 2) A tampon injury 3) Primary syphilis 4) Chlamydia

4) Chlamydia The correct response is "4" because these clinical manifestations are typical of a chlamydia infection (postcoital bleeding, dysuria, frequency, vaginal discharge, cervical tenderness with easily induced bleeding). Response "1" is incorrect because it would be rare to have cervical cancer at such a young age and the symptoms presented are not suggestive of this diagnosis. Response "2" is incorrect because the presenting symptoms are not suggestive of a cervical injury. Response "3" is incorrect because primary syphilis would present with a chancre lesion at the site where the bacteria entered the body, typically the vulva.

The nurse's discharge teaching plan for the woman with PID should reinforce which of the following potentially life-threatening complications? 1) Involuntary infertility 2) Chronic pelvic pain 3) Depression 4) Ectopic pregnancy

4) Ectopic pregnancy A ruptured tubal pregnancy secondary to an ectopic pregnancy can cause life-threatening hypovolemic shock. Without immediate surgical intervention, death can result. 1 is incorrect: involuntary infertility may be emotionally traumatic, but it is not life-threatening. 2 is incorrect: chronic pelvic pain secondary to adhesions is unpleasant but typically isn't life-threatening. 3 is incorrect: depression may be caused by the chronic pain or involuntary infertility but is not life-threatening.

Which instruction should be given to a woman newly diagnosed with genital herpes? 1) Have your partner use a condom when lesions are present. 2) Obtain a Papanicolaou (Pap) test every 3 years. 3) Use a water-soluble lubricant for relief of pruritus. 4) Limit stress and emotional upset as much as possible.

4) Limit stress and emotional upset as much as possible Stress, anxiety, and emotional upset seem to predispose a client to recurrent outbreaks of genital herpes. Sexual intercourse should be avoided during outbreaks, and a condom should be used between outbreaks; it is not known whether the virus can be transmitted at this time. During an outbreak, creams and lubricants should be avoided because they may prolong healing. Because a relationship has been found between genital herpes and cervical cancer, a Pap test is recommended every year.

A pregnant client's last normal menstrual period was on August 10. Using Nagele rule, the nurse calculates that her estimated due date (EDD) will be which of the following? 1) June 23 2) July 10 3) July 30 4) May 17

4) May 17 Using Nagele rule, 3 months are subtracted and 7 days are added, plus 1 year from the date of the last menstrual period.

Practicing good oral hygiene is important for all women throughout their pregnancy. As a nurse providing anticipatory guidance for pregnant women, what condition can result from periodontal disease if good dental care isn't practiced? 1) Post-dates pregnancy 2) Large for gestational age infant 3) Advanced reproductive cancer 4) Preterm or low-birth-weight infant

4) Preterm or low-birth-weight infant Infections within the mouth caused by poor oral hygiene spread into the maternal circulatory system and cross into the fetal circulation which inhibits growth and development of the fetus. Their uterine environment is compromised and early birth is a risk. Response "1" is incorrect due to maternal infections which tend to create a hostile environment resulting in an early birth, not a prolonged gestation. Response "2" is incorrect because just the opposite happens with the fetal growth becoming compromised due to infections. Response "3" is incorrect because research has not established an association between periodontal disease and the etiology of any reproductive cancer yet.

The healthcare provider is reviewing the results of a non-stress test (NST). In a 20 minute period, the test shows 2 fetal heart rate (FHR) accelerations above baseline that last 15 seconds. The FHR is 120 beats/minute, and the variability is average. How will the healthcare provider interpret this NST? Choose 1 answer: 1) Nonreactive 2) Positive 3) Negative 4) Reactive

4) Reactive The NST assesses fetal well-being by measuring fetal heart rate (FHR) variability and response to fetal movement. Decreased variability and lack of accelerations are associated with fetal hypoxia and acidosis. It can also be caused by fetal sleep, so attempts are made to wake the fetus up before determining the results of the NST. Average FHR variability and increases in FHR (accelerations) in response to fetal movement are considered reassuring signs, and a reactive NST. Remember NNN: Non-reactive Non-stress is Not good.

A woman with a history of pelvic inflammatory disease (PID) presents to the urgent care clinic with a report of dizziness, sharp abdominal pain, and scant dark red vaginal discharge. She has missed two menstrual periods and thinks she might be pregnant. A pregnancy test confirms her pregnancy. These assessment findings indicate the woman is experiencing which of the following problems? Choose 1 answer: 1) Placenta previa 2) Gestational trophoblastic disease 3) Placenta abruption 4) Ruptured ectopic pregnancy

4) Ruptured ectopic pregnancy PID can damage the reproductive organs so they don't function properly. Inflammation creates a blockage in the normal flow from the ovaries to the uterus. An ectopic pregnancy occurs when the fertilized egg is implanted outside the uterine cavity. As the embryo enlarges, rupture occurs, causing pain and bleeding.

For a woman, what is the most serious long term outcome of an infection resulting from exposure to the N. gonorrhea bacteria? 1) Post intercourse bleeding 2) Bartholin gland abscesses 3) Dysuria 4) Sterility

4) Sterility There may be infections of the uterus and development of acute or chronic infection of the fallopian tubes (i.e., salpingitis), with ultimate scarring and sterility. The other options are less serious in nature.

Which of the following fish should be limited in a pregnant woman's diet because of the high mercury content? 1) Salmon 2) Cod 3) Shrimp 4) Sword fish

4) Sword fish Sword fish typically contains high levels of mercury when compared to other fish species which should be avoided during pregnancy. Responses "1," "2," and "3" are fish that have been assessed as having low levels of mercury.

Anticipatory guidance regarding sexual activity during pregnancy includes which of the following? Select all that apply: 1) Sexual activity is contraindicated throughout pregnancy 2) Most women don't desire intimacy after the first trimester 3) Sexual activity may continue up until the end of the second trimester 4) Sexual intercourse is prohibited if a history of preterm labor exists 5) Women's sexual desire may change throughout the pregnancy 6) Couples can try a variety of positions of comfort during pregnancy

5) Women's sexual desire may change throughout the pregnancy 6) Couples can try a variety of positions of comfort during pregnancy Sexual desire changes throughout pregnancy based on hormones, energy levels, relationship, body image, fears of hurting the fetus, and cultural beliefs. Various positions of comfort for the woman are usually tried for most couples desiring intimacy. Responses "A" through "D" are incorrect because unless there is a medical reason why sexual intercourse isn't permitted, sexual activity may be continued throughout a healthy pregnancy.

A nurse is describing the different types of regional analgesia and anesthesia for labor to a group of pregnant women. Which statement by the group indicates that the teaching was successful? A) "We can get up and walk around after receiving combined spinal-epidural analgesia." B) "Higher anesthetic doses are needed for patient-controlled epidural analgesia. C) "A pudendal nerve block is highly effective for pain relief in the first stage of labor." D) "Local infiltration using lidocaine is an appropriate method for controlling contraction pain."

A) "We can get up and walk around after receiving combined spinal-epidural analgesia." When compared with traditional epidural or spinal analgesia, which often keeps the woman lying in bed, combined spinal-epidural analgesia allows the woman to ambulate ("walking epidural"). Patient-controlled epidural analgesia provides equivalent analgesia with lower anesthetic use, lower rates of supplementation, and higher client satisfaction. Pudendal nerve blocks are used for the second stage of labor, an episiotomy, or an operative vaginal birth with outlet forceps or vacuum extractor. Local infiltration using lidocaine does not alter the pain of uterine contractions, but it does numb the immediate area of the episiotomy or laceration. (pg. 467)

Which of the following is a priority when caring for a woman during the fourth stage of labor? A) Assessing the uterine fundus B) Offering fluids as indicated C) Encouraging the woman to void D) Assisting with perineal care

A) Assessing the uterine fundus During the fourth stage of labor, a priority is to assess the woman's fundus to prevent postpartum hemorrhage. Offering fluids, encouraging voiding, and assisting with perineal care are important but not an immediate priority. (pg. 483)

A client's membranes spontaneously ruptured, as evidenced by a gush of clear fluid with a contraction. Which of the following would the nurse do next? A) Check the fetal heart rate. B) Perform a vaginal exam. C) Notify the physician immediately. D) Change the linen saver pad.

A) Check the fetal heart rate. When membranes rupture, the priority focus is on assessing fetal heart rate first to identify a deceleration, which might indicate cord compression secondary to cord prolapse. A vaginal exam may be done later to evaluate for continued progression of labor. The physician should be notified, but this is not a priority at this time. Changing the linen saver pad would be appropriate once the fetal status is determined and the physician has been notified. (pg. 446)

The nurse notes persistent early decelerations on the fetal monitoring strip. Which of the following would the nurse do next? A) Continue to monitor the FHR because this pattern is benign. B) Perform a vaginal exam to assess cervical dilation and effacement. C) Stay with the client while reporting the finding to the physician. D) Administer oxygen after turning the client on her left side.

A) Continue to monitor the FHR because this pattern is benign. Early decelerations are not indicative of fetal distress and do not require intervention. Therefore, the nurse would continue to monitor the fetal heart rate pattern. They are most often seen during the active stage of any normal labor, during pushing, crowning, or vacuum extraction. They are thought to be a result of fetal head compression that results in a reflex vagal response with a resultant slowing of the FHR during uterine contractions. There is no need to perform a vaginal exam, report the finding to the physician, or administer oxygen. (pg. 455)

A woman has just entered the second stage of labor. The nurse would focus care on which of the following? A) Encouraging the woman to push when she has a strong desire to do so B) Alleviating perineal discomfort with the application of ice packs C) Palpating the woman's fundus for position and firmness D) Completing the identification process of the newborn with the mother

A) Encouraging the woman to push when she has a strong desire to do so During the second stage of labor, nursing interventions focus on motivating the woman, encouraging her to put all her efforts toward pushing. Alleviating perineal discomfort with ice packs and palpating the woman's fundus would be appropriate during the fourth stage of labor. Completing the newborn identification process would be appropriate during the third stage of labor. (pg. 475)

The nurse is performing Leopold's maneuvers to determine fetal presentation, position, and lie. Which action would the nurse do first? A) Feel for the fetal buttocks or head while palpating the abdomen. B) Feel for the fetal back and limbs as the hands move laterally on the abdomen. C) Palpate for the presenting part in the area just above the symphysis pubis. D) Determine flexion by pressing downward toward the symphysis pubis.

A) Feel for the fetal buttocks or head while palpating the abdomen. The first maneuver involves feeling for the buttocks and head. Next the nurse palpates on which side the fetal back is located. The third maneuver determines presentation and involves palpating the area just above the symphysis pubis. The final maneuver determines attitude and involves applying downward pressure in the direction of the symphysis pubis. (pg. 448)

A nurse palpates a woman's fundus to determine contraction intensity. Which of the following would be most appropriate for the nurse to use for palpation? A) Finger pads B) Palm of the hand C) Finger tips D) Back of the hand

A) Finger pads To palpate the fundus for contraction intensity, the nurse would place the pads of the fingers on the fundus and describe how it feels. Using the finger tips, palm, or back of the hand would be inappropriate. (pg. 447)

A client with a history of HSV-2 infection asks the nurse about future sexual activity. Which response would be most appropriate? a) "Inform all potential sexual partners about the infection, even if it is inactive." b) "Use a condom during sexual activity if the infection becomes active again." c) "Refrain from all sexual activity until you don't have another outbreak for a year." d) "If the infection has healed, you probably don't have to use a condom."

A) Inform all potential sexual partners about the infection, even if it is inactive. The nurse should advise the client to inform all potential sexual partners of the HSV infection, even if it is in an inactive state. The nurse should also advise the client to use a condom during sexual activity, even if the disease is dormant, and to avoid sexual contact if the infection is active. Condoms do not protect skin and mucous membranes left exposed.

A woman in labor received an opioid close to the time of birth. The nurse would assess the newborn for which of the following? A) Respiratory depression B) Urinary retention C) Abdominal distention D) Hyperreflexia

A) Respiratory depression Opioids given close to the time of birth can cause central nervous system depression, including respiratory depression, in the newborn, necessitating the administration of naloxone. Urinary retention may occur in the woman who received neuraxial opioids. Abdominal distention is not associated with opioid administration. Hyporeflexia would be more commonly associated with central nervous system depression due to opioids. (pg. 464)

After teaching a group of students about fetal heart rate patterns, the instructor determines the need for additional teaching when the students identify which of the following as indicating normal fetal acid-base status? (Select all that apply.) A) Sinusoidal pattern B) Recurrent variable decelerations C) Fetal bradycardia D) Absence of late decelerations E) Moderate baseline variability

A) Sinusoidal pattern B) Recurrent variable decelerations C) Fetal bradycardia Predictors of normal fetal acid-base status include a baseline rate between 110 and 160 bpm, moderate baseline variability, and absences of later or variable decelerations. Sinusoidal pattern, recurrent variable decelerations, and fetal bradycardia are predictive of abnormal fetal acid-base status. (pg. 453)

Which position would be most appropriate for the nurse to suggest as a comfort measure to a woman who is in the first stage of labor? (Select all that apply.) A) Walking with partner support B) Straddling with forward leaning over a chair C) Closed knee-chest position D) Rocking back and forth with foot on chair E) Supine with legs raised at a 90-degree angle

A) Walking with partner support B) Straddling with forward leaning over a chair D) Rocking back and forth with foot on chair Positioning during the first stage of labor includes walking with support from the partner, side-lying with pillows between the knees, leaning forward by straddling a chair, table, or bed or kneeling over a birthing ball, lunging by rocking weight back and forth with a foot up on a chair or birthing ball or an open knee-chest position. (pg. 475)

The nurse should initially implement which intervention when a nulliparous woman telephones the hospital to report that she is in labor. a) Ask the woman to describe why she believes that she is in labor b) Tell the woman to stay home until her membranes rupture c) Arrange for the woman to come to the hospital for labor evaluation d) Emphasize that food and fluid should stop or be light

Ask the woman to describe why she believes that she is in labor The nurse needs further information to assist in determining if the woman is in true or false labor. She will need to ask the patient questions to seek further assessment and triage information. Having her wait until membranes rupture may be dangerous, as she may give birth before reaching the hospital. She should continue fluid intake until it is determined whether or not she is in labor. She may be in false labor, and more information should be obtained before she is brought to the hospital.

The nurse understands that the maternal uterus should be at what location at 20 weeks' gestation? a) At the level of the symphysis pubis b) At the level near the bottom of the sternum c) At the level of the umbilicus d) Three finger-breadths above the umbilicus

At the level of the umbilicus By 20 weeks' gestation, the uterus is at about the level of the umbilicus; by 36 weeks', it nears the bottom of the sternum.

What instruction should a nurse offer to a pregnant client or a client who wishes to become pregnant to help her avoid exposure to teratogenic substances? a) Avoid medications b) Avoid intake of coffee c) Eat a well-balanced diet d) Maintain personal hygiene

Avoid medications The nurse should instruct a client who is pregnant or one who wants to conceive to avoid medications to enable the client to avoid exposure to any kind of teratogenic substance. Eating a well-balanced diet and maintaining personal hygiene, though important during pregnancy, will not prevent a client's exposure to teratogenic substances. Coffee is not a teratogenic substance so the client need not avoid coffee. However, coffee is not recommended during pregnancy because it may increase the risk of spontaneous abortion.

A client is diagnosed with pelvic inflammatory disease (PID). When reviewing the client's medical record, which of the following would the nurse expect to find? (Select all that apply.) A) Oral temperature of 100.4 degrees F B) Dysmenorrhea C) Dysuria D) Lower abdominal tenderness E) Discomfort with cervical motion F) Multiparity

B) Dysmenorrhea C) Dysuria D) Lower abdominal tenderness E) Discomfort with cervical motion

A nurse is completing the assessment of a woman admitted to the labor and birth suite. Which of the following would the nurse expect to include as part of the physical assessment? (Select all that apply.) A) Current pregnancy history B) Fundal height measurement C) Support system D) Estimated date of birth E) Membrane status F) Contraction pattern

B) Fundal height measurement E) Membrane status F) Contraction pattern As part of the admission physical assessment, the nurse would assess fundal height, membrane status and contractions. Current pregnancy history, support systems, and estimated date of birth would be obtained when collecting the maternal health history. (pg. 471)

A woman's amniotic fluid is noted to be cloudy. The nurse interprets this finding as which of the following? A) Normal B) Possible infection C) Meconium passage D) Transient fetal hypoxia

B) Possible infection Amniotic fluid should be clear when the membranes rupture, either spontaneously or artificially through an amniotomy (a disposable plastic hook [Amnihook] is used to perforate the amniotic sac). Cloudy or foul-smelling amniotic fluid indicates infection. Green fluid may indicate that the fetus has passed meconium secondary to transient hypoxia, prolonged pregnancy, cord compression, intrauterine growth restriction, maternal hypertension, diabetes, or chorioamnionitis; however, it is considered a normal occurrence if the fetus is in a breech presentation. (pg. 447)

Why is the first prenatal visit usually the longest prenatal visit? a) Extensive patient teaching is done b) Lab tests are performed c) Baseline data is collected d) A pelvic exam with Pap smear is performed

Baseline data is collected The first prenatal visit is usually the longest because the baseline data to which all subsequent assessments are compared are obtained at this visit.

After teaching a class on the various structures formed by the embryonic membranes, the nurse determines that the teaching was successful when the class identifies which structure as being formed by the mesoderm? a) Stomach b) Ears c) Lungs d) Bones Bones

Bones The endoderm forms the structures of the respiratory system. The mesoderm forms the structures of the skeletal system. The ectoderm forms the structures of the special senses. The endoderm forms the structures of the digestive system.

When applying the ultrasound transducers for continuous external electronic fetal monitoring, at which location would the nurse place the transducer to record the FHR? A) Over the uterine fundus where contractions are most intense B) Above the umbilicus toward the right side of the diaphragm C) Between the umbilicus and the symphysis pubis D) Between the xiphoid process and umbilicus

C) Between the umbilicus and the symphysis pubis The ultrasound transducer is positioned on the maternal abdomen in the midline between the umbilicus and the symphysis pubis. The tocotransducer is placed over the uterine fundus in the area of greatest contractility. (pg. 452)

A nurse is assisting with the delivery of a newborn. The fetal head has just emerged. Which of the following would be done next? A) Suctioning of the mouth and nose B) Clamping of the umbilical cord C) Checking for the cord around the neck D) Drying of the newborn

C) Checking for the cord around the neck Checking for the cord around the neck once the fetal head has emerged, the primary care provider explores the fetal neck to see if the umbilical cord is wrapped around it. If it is, the cord is slipped over the head to facilitate delivery. Then the health care provider suctions the newborn's mouth first (because the newborn is an obligate nose breather) and then the nares with a bulb syringe to prevent aspiration of mucus, amniotic fluid, or meconium. Finally the umbilical cord is double-clamped and cut between the clamps. The newborn is placed under the radiant warmer, dried, assessed, wrapped in warm blankets and placed on the woman's abdomen for warmth and closeness. (pg. 480)

When assessing fetal heart rate, the nurse finds a heart rate of 175 bpm, accompanied by a decrease in variability and late decelerations. Which of the following would the nurse do next? A) Have the woman change her position. B) Administer oxygen. C) Notify the health care provider. D) Continue to monitor the pattern every 15 minutes.

C) Notify the health care provider. Fetal tachycardia as evidenced by a fetal heart rate greater than 160 bpm accompanied by a decrease in variability and late decelerations is an ominous sign indicating the need for prompt intervention. The health care provider should be notified immediately and then measures should be instituted such as having the woman lie on her side and administering oxygen. In this instance, monitoring should be continuous to detect any further changes and evaluate the effectiveness of interventions. (pg. 453)

Which body system is most affected throughout the embryonic and fetal period by teratogens?

Central nervous system

A pregnant patient is concerned that the baby is going to drown in the uterus because of the fluid. What should the nurse respond about fetal respiration? A) "You are breathing for the baby." B) "The baby's breathing is very minor until delivery." C) "The baby's lungs can accommodate all of the fluid." D) "Oxygen is provided to the baby through the placenta."

D) "Oxygen is provided to the baby through the placenta. Fetal circulation differs from extrauterine circulation because the fetus derives oxygen and excretes carbon dioxide not from gas exchange in the lung but from exchange in the placenta. The baby's lungs are not functioning in utero so the response that the baby's breathing is very minor until delivery and the baby's lungs being able to accommodate the fluid are incorrect. The patient is not "breathing for the baby."

A woman in labor has chosen to use hydrotherapy as a method of pain relief. Which statement by the woman would lead the nurse to suspect that the woman needs additional teaching? A) "The warmth and buoyancy of the water has a nice relaxing effect." B) "I can stay in the bath for as long as I feel comfortable." C) "My cervix should be dilated more than 5 cm before I try using this method." D) "The temperature of the water should be at least 105°F."

D) "The temperature of the water should be at least 105°F." Hydrotherapy is an effective pain relief method. The water temperature should not exceed body temperature. Therefore, a temperature of 105° F would be too warm. The warmth and buoyancy have a relaxing effect and women are encouraged to stay in the bath as long as they feel comfortable. The woman should be in active labor with cervical dilation greater than 5 cm. (pg. 458)

A nurse is teaching a young adult woman about sexually transmitted infections. The nurse determines that the teaching was successful when the woman identifies which infection as being characterized by genital ulcers? a) chlamydia B) bacterial vaginosis c) gonorrhea d) genital herpes simplex

D) genital herpes simplex Genital herpes simplex is a sexually transmitted infection associated with genital ulcers. Gonorrhea and chlamydia are associated with cervicitis. Bacterial vaginosis is associated with a vaginal discharge.

When planning the care of a woman in the active phase of labor, the nurse would anticipate assessing the fetal heart rate at which interval? A) Every 2 to 4 hours B) Every 45 to 60 minutes C) Every 15 to 30 minutes D) Every 10 to 15 minutes

During the active phase of labor, FHR is monitored every 15 to 30 minutes. FHR is assessed every 30 to 60 minutes during the latent phase of labor. The woman's temperature is typically assessed every 4 hours during the first stage of labor and every 2 hours after ruptured membranes. Blood pressure, pulse, and respirations are assessed every hour during the latent phase and every 30 minutes during the active and transition phases. Contractions are assessed every 30 to 60 minutes during the latent phase and every 15 to 30 minutes during the active phase, and every 15 minutes during transition. (pg. 473)

A 38-year-old client presents to the clinic desiring to get pregnant. She reports she had a tubal ligation in her early 20s after two babies and a divorce. After learning that the client recently underwent a reversal of the tubal ligation, the nurse will warn the client of which potential risk?

Ectopic pregnancy

A client in her second trimester of pregnancy has developed varicose veins and experiences leg cramps. Which of the following suggestions would be most appropriate? a) Increase intake of folic acid b) Elevate legs while sitting c) Increase intake of calcium d) Perform aerobic exercises

Elevate legs while sitting The nurse should encourage the client to elevate her legs while sitting; this will prevent pooling and engorgement of veins in the lower extremities. Aerobic exercises do not help in preventing varicose veins. Folic acid intake is recommended in the first trimester to prevent congenital abnormalities. Increasing the intake of calcium helps in strengthening bones.

Which of the following findings from a woman's initial prenatal assessment would be considered a possible complication of pregnancy that requires reporting to a physician for management? a) Nasal congestion and swollen nasal membranes b) Episodes of double vision c) Palpitations when lying on her back d) Increased lumbar curvature

Episodes of double vision Difficulty with vision can occur from cerebral edema or is a symptom of hypertension of pregnancy.

A woman who is 4 months pregnant notices frequent heart palpitations and leg cramps. She is anxious to learn how to alleviate these. Which of the following nursing diagnoses would best apply to her? a) Risk for ineffective breathing pattern related to pressure of the growing uterus b) Impaired urinary elimination related to inability to excrete creatine from her muscles c) Health-seeking behaviors related to ways to relieve discomforts of pregnancy d) Pain related to severe complications of pregnancy

Health-seeking behaviors related to ways to relieve discomforts of pregnancy Health-seeking behaviors is a diagnosis used to describe patients who are actively interested in learning ways to improve their health.

A nurse assesses a 32-year-old primigravida client with twin gestation in her second trimester. The client reports constipation from iron supplements. Which condition should the nurse assess for in this client as a result of the constipation? a) Hemorrhoids b) Gastric ulcer c) Ptyalism d) Thrombophlebitis

Hemorrhoids The nurse should assess this client for hemorrhoids. Constipation, is a common problem during pregnancy, especially in clients who take iron supplements and hemorrhoids may develop because of the pressure on the venous structures from straining to have a bowel movement. Gastric ulcers may cause bleeding and would be a reason for taking iron supplements. Clients who are placed on bedrest during pregnancy are at a very high risk for development of thrombophlebitis. Ptyalism or excessive salivation may occur in the first trimester.

Leah is 28 weeks pregnant. In preparing for discomforts that occur during the final trimester of pregnancy, you would teach her about? a) Eating a well-balanced diet to prevent anemia b) Increased shortness of breath and dyspnea before lightening c) Avoid exercise to prevent varicosities d) Good oral hygiene to decrease ptyalism

Increased shortness of breath and dyspnea before lightening As the fetus grows inside the mother, there is more pressure on the diaphragm and more difficulty breathing, and episodes of dyspnea may occur. This tends to decrease with lightening, when the fetus drops. Eating a well balanced diet, oral hygiene, and exercise should be done throughout the entire pregnancy.

The nurse is explaining the procedure for ultrasound to a pregnant client and is discussing the amniotic fluid and how they will determine if there is too much or too little fluid. The client asks about the purpose of the amniotic fluid. What should the nurse tell the client? 1) It provides unrestricted movement. 2) It provides symmetrical growth. 3) It produces hormones. 4) It is for the physical protection of the growing fetus. 5) It regulates temperature.

It is for the physical protection of the growing fetus. It regulates temperature. It provides unrestricted movement. It provides symmetrical growth. Amniotic fluid fills the amniotic cavity. It serves four main functions for the fetus: physical protection, temperature regulation, provision of unrestricted movement, and symmetrical growth.

If a pregnant woman's estimated date of delivery (EDD) is April 23, what was the first day of her last menstrual period (LMP), according to Nagele's rule? a) July 16 b) July 19 c) July 23 d) July 13

July 16 According to Nagele's rule, the last menstrual period was July 16th. Take the LMP and add 7 days and subtract 3 months; if finding the LMP from the EDD, subtract 7 days and add 3 months.

A pregnant patient is directed to perform a daily fetal movement count. What should the nurse instruct the patient about this count? Select all that apply.

Lie down to do the count after eating a meal. Count fetal movements until a total of 10 are counted and record the time.

A woman is taking vaginal progesterone suppositories during her first trimester because her body does not produce enough of it naturally. She asks the nurse what function this hormone has in her pregnancy. The nurse should mention which of the following as the primary function of progesterone? a) Ensures the corpus luteum of the ovary continues to produce estrogen b) Contributes to mammary gland development c) Maintains the endometrial lining of the uterus during pregnancy d) Regulates maternal glucose, protein, and fat levels

Maintains the endometrial lining of the uterus during pregnancy Progesterone is necessary to maintain the endometrial lining of the uterus during pregnancy. It is human chorionic gonadotropin (hCG) that acts to ensure the corpus luteum of the ovary continues to produce estrogen and progesterone. Estrogen contributes to mammary gland development, and human placental lactogen regulates maternal glucose, protein, and fat levels.

A client in her third trimester of pregnancy visits the healthcare center and asks why she is constipated. Which of the following would the nurse include as the most likely cause when responding to the client? a) Pressure on intestine by the growing fetus b) Pressure of fetal head on the bladder c) Engorgement of veins by the weight of the uterus d) Relaxation of cardioesophageal sphincter

Pressure on intestine by the growing fetus The nurse should explain that constipation often occurs during the third trimester because the growing fetus exerts pressure on the intestine. Engorgement of veins by the weight of the uterus causes varicosities. Pressure of the fetal head on the bladder increases the frequency of urination. Relaxation of the cardioesophageal sphincter causes heartburn.

The nurse is teaching a patient in the first trimester of pregnancy about the importance of folic acid in the diet and how folic acid supplements might be beneficial. For which reason is the nurse teaching the patient about this vitamin?

Prevents neural tube disorders in the developing fetus

Pregnancy tests analyze for the presence of human chorionic gonadotropin (HCG) hormone. The nurse understands the function of this hormone as which of the following? a) Prolong the life of the corpus luteum b) Permit the uterus to increase in size c) Ensure chorionic somatomammotropin hormone production d) Ensure oxygen transport across the placenta

Prolong the life of the corpus luteum The corpus luteum is responsible for producing progesterone until this function is assumed by the placenta. HCG is a "fail-safe" mechanism to prolong the life of the corpus luteum and ensure progesterone production. Somatomammotropin is a growth hormone. The uterine growth is in response to estrogen and other growth factors. Oxygen transport occurs via blood flow from the mother.

A nurse is coaching a woman during the second stage of labor. Which of the following should the nurse encourage the client to do at this time? a) Hold her breath while pushing during contractions b) Begin pushing as soon as the cervix has dilated to 8 cm c) Push with contractions and rest between them d) Pant while she pushes

Push with contractions and rest between them Make sure the woman pushes with contractions and rests between them. Holding the breath during a contraction could cause a Valsalva maneuver or temporarily impede blood return to her heart because of increased intrathoracic pressure, which could then also interfere with blood supply to the uterus. It is important for women to understand they should not bear down with their abdominal muscles to push until the cervix is fully dilated, which is 10 cm, not 8 cm. Panting limits the ability to push and is to be encouraged only when it is desirable to delay labor, such as when a nuchal cord is present.

A pregnant woman comes to the emergency department because she thinks she is in labor. The nurse determines that the client is in true labor when assessment of contractions reveals which of the following? a) Lasting about 20 to 30 seconds b) Occurring in an irregular pattern c) Radiating to the front of the abdomen from the back d) Slowing when the woman changes position

Radiating to the front of the abdomen from the back Contractions that begin in the back and then radiate to the front are typical of true labor. Contractions that slow when a woman walks or changes position suggest false labor, as do irregular contractions. Contractions lasting 30 seconds or less commonly suggest Braxton-Hicks contractions and are associated with false labor.

During pregnancy, which of the following should the expectant mother reduce or avoid? a. Raw meat or uncooked shellfish b. Fresh, washed fruits and vegetables c. Whole grains d. Protein and iron from meat sources

Raw meat or uncooked shellfish

A client comes to the clinic with concerns about her pregnancy. She is in her first trimester and is now experiencing moderate abdominal pain on the right side. What would be the nurse's first action?

Recommend an abdominal ultrasound to the doctor since this may be ectopic pregnancy.

A pregnant woman calls her provider's office to report she thinks she is in labor. The patient reports contractions have been at these times: 12:05, 12:10, 12:15, and 12:20. What information is gathered based on this data? a) The contractions are increasing in duration and frequency b) The frequency of the contractions is every 5 minutes c) Nothing useful about the client's contractions d) The duration of the contractions is every 5 minutes

The frequency of the contractions is every 5 minutes Based on the information, the nurse knows the contractions are regular and every 5 minutes apart. This is the only data gathered based on the information given, but it is very useful to the provider.

The nurse is developing a teaching plan for a client diagnosed with genital herpes simplex (HSV). Which would the nurse include? Select all that apply.

The management of genital herpes includes antiviral therapy. The need to use good hand washing technique to prevent spread. Educate the client to abstain from sexual activity until HSV lesions resolve. The goal is for recurrences to be less frequent over time.

A pregnant woman undergoing amniocentesis asks her nurse why the baby needs this fluid. What would be an accurate response from the nurse? a) "Amniotic fluid cushions your baby to prevent injury." b) "Amniotic fluid keeps the fetus from moving freely inside it to prevent injury." c) "Amniotic fluid provides fetal blood circulation." d) "Amniotic fluid supplies the food your baby needs to grow."

a) "Amniotic fluid cushions your baby to prevent injury." Explanation: The amniotic fluid, kept inside the amnion, cushions the fetus against injury, regulates temperature, and allows the fetus to move freely inside it, which allows normal musculoskeletal development of the fetus. The woman's blood supplies food to—and carries wastes away from—the fetus. The placenta supplies the developing organism with food and oxygen; then the umbilical cord connects the fetal blood vessels contained in the villi of the placenta with those found within the fetal body.

A 20 year-old male has been diagnosed with a chlamydial infection, and his primary care provider is performing teaching in an effort to prevent the client from infecting others in the future. Which of the following statements by the client demonstrates the best understanding of his health problem? a) "Even if I spread it to someone else, there's a good chance she won't have any symptoms or know she has it." b) "Each of the 3 stages of the disease seems to be worse than the previous one." c) "Either me or a partner could end up with an eye infection from chlamydia that could make us blind." d) "Even though I couldn't end up sterile, a woman that I give it to certainly could."

a) "Even if I spread it to someone else, there's a good chance she won't have any symptoms or know she has it."

The nurse is interviewing parents after their newborn was diagnosed with a genetic disorder. Which statements by the mother is associated with risk factors of genetic disorders? Select all that apply. a) "My sister's baby was born with trisomy 18." b) "He is our first child." c) "My husband is 55 years old." d) "Our alpha-fetoprotein came back negative when I was 18 weeks pregnant." e) "Our obstetrician told us that I wasn't making enough amniotic fluid during this pregnancy."

a) "My sister's baby was born with trisomy 18." c) "My husband is 55 years old." e) "Our obstetrician told us that I wasn't making enough amniotic fluid during this pregnancy." The following are risk factors for genetic disorders: oligohydramnios, paternal age over 50, a family history of genetic disorders, positive alpha-fetoprotein test, and multiple births.

When she discovers that she is unexpectedly pregnant, a patient calls the nurse in a panic. She reports that although she does not drink heavily, the night she must have gotten pregnant she was at a wedding and had drunk quite a lot. The day after the wedding she had taken several acetaminophens. Since that day 3 weeks earlier, she has also had a nightly glass of wine with dinner. She asks if she has caused irreversible damage to the fetus. What is an appropriate response for the nurse to make? a) "You didn't know you were pregnant, so you haven't done anything wrong. The fetus isn't exposed to the mother's blood until after it implants, so the wedding's not an issue. But, don't have any more alcohol while you're pregnant." b) "Why did you have unprotected sex if you had been drinking?" c) "You didn't know you were pregnant, so you haven't done anything intentionally wrong. But alcohol is very damaging to the growing fetus, so you'd better be sure to stop drinking. Do you need any support for that?" d) "Exposure to alcohol can cause facial deformities, low birth weights, and underdeveloped brains. The wedding night isn't an issue because the fetus isn't exposed to the mother's blood at first, but I hope this last week of drinking hasn't caused any problems."

a) "You didn't know you were pregnant, so you haven't done anything wrong. The fetus isn't exposed to the mother's blood until after it implants, so the wedding's not an issue. But, don't have any more alcohol while you're pregnant." The fetus was not developing during the initial night of drinking. Now that the fetus is implanted and developing, the fetus would be exposed to the influences of the mother. Alcohol use should be stopped now that she knows she is pregnant, but reassure her that the limited amounts she has consumed will most likely not have an adverse effect on the fetus. As a practitioner, you should not question her choice of drinking and having unprotected sex.

A client in her second trimester of pregnancy arrives at a health care facility complaining of heartburn. What instructions should the nurse offer to help the client deal with heartburn? Select all that apply. a) Avoid overeating b) Limit consumption of food before bedtime c) Sleep in a semi-Fowler's position d) Avoid use of antacids e) Consume lots of liquids before bedtime

a) Avoid overeating b) Limit consumption of food before bedtime c) Sleep in a semi-Fowler's position When caring for a pregnant client with heartburn, the nurse should instruct the client to limit consuming foods before bedtime. The nurse should also instruct the client to sleep in a semi-Fowler's position and to avoid overeating. The nurse need not instruct the client to avoid the use of antacids. On the contrary, antacids are known to be useful for heartburn even during pregnancy, so the nurse need not instruct the client to avoid them. The nurse should not instruct the client to consume lots of fluids before bedtime. Along with food, even fluids should be limited before bedtime.

When teaching a group of nursing students about the different types of pelvis, the nurse describes which of the following as the features of a gynecoid pelvis? Select all that apply. a) Dull ischial spines b) Straight sacrum c) Convergent side walls d) Round-shaped inlet e) Wide pubic arch

a) Dull ischial spines d) Round-shaped inlet e) Wide pubic arch The features of a gynecoid pelvis are oval-shaped inlet, dull ischial spines, and wide pubic arch. The birth of a baby is easiest in a gynecoid pelvis. Straight sacrum and convergent side walls are seen in an android pelvis.

One specific vitamin is known to prevent up to 70 percent of birth defects of the central nervous system, called neural tube defects. Which vitamin should all woman who are at risk for getting pregnant have daily in their diet? a) Folic Acid b) Vitamin A c) Iodine d) Zinc

a) Folic Acid

To deliver her infant, a woman is asked to push with contractions. Which of the following is the most effective and safest pushing technique to teach her? a) Head elevated, grasping knees, breathing out b) Squatting while holding her breath c) Lying supine with legs in lithotomy stirrups d) Lying on side, arms grasped on abdomen

a) Head elevated, grasping knees, breathing out An important point is to be certain the woman does not hold her breath, as this puts pressure on the vena cava, reducing blood return.

Pregnancy tests (both urine and blood) measure levels of which hormone to validate the existence of pregnancy? a) Human chorionic gonadotropin (hCG) b) Estrogen c) Progesterone d) Aldosterone

a) Human chorionic gonadotropin (hCG)

A client comes to the prenatal clinic for a follow-up examination. When assessing the breasts, which of the following would the nurse expect to find? Select all that apply. a) Hyperpigmentation of the nipple b) Pallor of the areolae c) Increased sensitivity d) Prominent veins e) Warmth

a) Hyperpigmentation of the nipple c) Increased sensitivity d) Prominent veins

A pregnant client has been diagnosed with gonorrhea. Which nursing interventions should be performed to prevent gonococcal ophthalmia neonatorum in the baby? a) Instill a prophylactic agent in the eyes of the newborn. b) Perform cesarean birth to prevent infection. c) Administer an antiretroviral syrup to the newborn. d) Administer cephalosporins to mother during pregnancy.

a) Instill a prophylactic agent in the eyes of the newborn. To prevent gonococcal ophthalmia neonatorum in the baby, the nurse should instill a prophylactic agent in the eyes of the newborn. Cephalosporins are administered to the mother during pregnancy to treat gonorrhea but not to prevent infection in the newborn. Performing a cesarean birth will not prevent gonococcal ophthalmia neonatorum in the newborn. An antiretroviral syrup is administered to the newborn only if the mother is human immunodeficiency virus-positive and will not help prevent gonococcal ophthalmia neonatorum in the baby.

A nurse is conducting prenatal classes and is reviewing the stages of fetal development. When discussing the pre-embryonic stage, what should the nurse inform the group about when this stage begins and ends? a) It begins at fertilization and lasts through the end of the second week after fertilization. b) It begins at 6 weeks after fertilization and ends at the 8th week after fertilization. c) It begins at 9 weeks after fertilization and ends at birth. d) It begins approximately 2 weeks after fertilization and ends at the conclusion of the eighth week after fertilization.

a) It begins at fertilization and lasts through the end of the second week after fertilization. The pre-embryonic stage begins at fertilization and lasts through the end of the second week after fertilization.

The obstetrical nurse knows that a woman's hormone levels change dramatically during pregnancy. Which of the following hormonal actions accurately represent these changes? Select all that apply. a) Maintaining the endometrium so that the embryo can implant b) Causing changes in the mother's metabolism so that nutrients are available for both c) Preparing the breasts for lactation, keeping the milk from coming in until birth occurs d) Relaxing the ligaments that connect the pelvic bones, allowing them to spread slightly e) Decreasing the blood supply to the gastrointestinal tract and slowing peristaltic waves f) Decreasing the mother's blood volume and red blood cell mass to increase oxygen

a) Maintaining the endometrium so that the embryo can implant b) Causing changes in the mother's metabolism so that nutrients are available for both c) Preparing the breasts for lactation, keeping the milk from coming in until birth occurs d) Relaxing the ligaments that connect the pelvic bones, allowing them to spread slightly

A nurse counsels a pregnant woman regarding her recommended daily allowance of calories. She advises her to obtain her carbohydrate calories from complex carbohydrates rather than simple carbohydrates. What is the best rationale for this guidance? a) More consistent regulation of glucose and insulin b) Greater fatty acid content c) Faster digestion of complex than simple carbohydrates d) Provision of a greater amount of calories per gram

a) More consistent regulation of glucose and insulin Advise women to obtain their carbohydrate calories from complex carbohydrates (cereals and grains) rather than simple carbohydrates (sugar and fruits) because complex carbohydrates are more slowly digested. Doing so will help regulate glucose and insulin levels more consistently. All carbohydrates contain roughly the same amount of calories per gram (4 kcal/g). Carbohydrates of any kind are not a significant source of fatty acids.

A client is in the first stage of labor and asks the nurse what type of pain she should expect at this stage. What is the nurse's most appropriate response? a) Pain from the dilation or stretching of the cervix b) Pressure on the lower back, buttocks, and thighs c) Hypoxia of the contracting uterine muscles d) Distention of the vagina and perineum

a) Pain from the dilation or stretching of the cervix In the first stage of labor, the primary source of pain is the dilation of the cervix. Hypoxia of the contracting uterine muscles, distension of the vagina and perineum, and pressure on the lower back, buttocks, and thighs may occur in the first stage but are more significantly associated with the second stage of labor.

The nurse explains to the pregnant patient that the placenta is important for many reasons, but one function it has is to secrete hormones to sustain the pregnancy. Which hormones does the nurse inform the patient that the placenta secretes? (Select all that apply.) a) Progesterone b) Testosterone c) Estrogen d) Human placental lactogen e) Human chorionic gonadotropin

a) Progesterone c) Estrogen d) Human chorionic gonadotropin e) Human placental lactogen The placenta secretes hormones that help to sustain the pregnancy. These include progesterone, estrogen, human placental lactogen, and human chorionic gonadotropin. Progesterone is necessary to maintain the nutrient-rich endometrial lining. It also functions to keep the myometrium quiet, so that contractions do not occur prematurely, leading to loss of the pregnancy. Estrogen functions to provide a rich blood supply to the decidua and placenta. The main function of human chorionic gonadotropin is to sustain the corpus luteum at the beginning of the pregnancy. The main function of hPL is to regulate glucose available for the fetus.

The client at 18 weeks' gestation states, "I feel a fluttering sensation, kind of like gas." The nurse understands that the client is describing what occurrence? a) Quickening b) Placenta previa c) Lightening d) Linea nigra

a) Quickening

A nurse is caring for a client in her third stage of labor. Which of the following would the nurse assess as indicating placental separation? Select all that apply. a) Renewed bearing down efforts by client b) Fresh gushing of blood from the vagina c) Falling downward of uterus in the abdomen d) A relaxed and distended uterus e) Umbilical cord descending lower down

a) Renewed bearing down efforts by client b) Fresh gushing of blood from the vagina e) Umbilical cord descending lower down The signs of placental separation include a fresh gush of blood from the vagina, lengthening of the umbilical cord, and renewed bearing-down efforts by the client. When the client is in her third stage of labor, these indicate placental separation. A rising upwards of the uterus and a well-contracted globular uterus are the other signs of placental separation. Falling downward of the uterus in the abdomen and a relaxed uterus are the signs of uterine atony.

The nurse midwife is performing a pelvic examination on a client who came to her following a positive home pregnancy test. The nurse checks the woman's cervix for the probable sign of pregnancy known as Goodell's sign. Which of the following describes this alteration? a) The cervix softens. b) The lower uterine segment softens. c) The cervix looks blue or purple when examined. d) The fundus enlarges.

a) The cervix softens.

A woman you care for in a prenatal clinic tells you that her pregnancy was unplanned and unwanted. At what point in pregnancy does the average woman change her mind about an unwanted pregnancy? a) When quickening occurs b) After the seventh month c) After lightening happens d) Around the third month

a) When quickening occurs

A 31 year-old male was diagnosed with genital herpes of the HSV-2 type 5 years ago. He is now broaching the subject with a woman he has recently formed a relationship with. Which of his statements is most accurate? a) "The worst case scenario is that you'll develop cold sores, since this is the type of herpes virus that I've got." b) "Even when I'm not having a recurrence, I could still pass the virus on to you." c) "If you've had cold sores when you were younger, it means that you've got antibodies against this type of herpes virus." d) "If you've been exposed to the herpes virus in the past, then there's no significant risk of reinfection."

b) "Even when I'm not having a recurrence, I could still pass the virus on to you."

A patient, 38 weeks gestation, pregnant with her first child calls the clinic and states "my baby is lower and it is more difficult to walk" and ask if she should come to the hospital to be checked. How should the nurse respond? a) "That is something we expect with a second or third baby, because it is your fist, you need to be checked." b) "The baby has dropped into the pelvis, your body and baby are getting ready for labor in the next few weeks." c) "The baby moved down into the pelvis, this means you will be in labor within 24 hours, wait for contractions to come to the hospital." d) "This is not normal unless you are in active labor, come to the hospital and be checked."

b) "The baby has dropped into the pelvis, your body and baby are getting ready for labor in the next few weeks." The baby can drop into the pelvis, an event termed lightening, and can happen for up to 2 weeks before the woman goes into labor. This is normal and does not require intervention.

A school health nurse is providing education to a group of adolescents regarding the proper procedure for male condom use. The nurse knows the teaching has been effective when which statement is made by a student? a) "It is important to put the condom on just before the penis is erect." b) "Withdraw the penis erect, holding the condom firmly against the penis." c) "Use only petroleum-based lubricants, such as body lotion or massage oil." d) "Ensure your condom is always available, so store condoms in your wallet."

b) "Withdraw the penis erect, holding the condom firmly against the penis." The teaching guidelines for proper condom use include: ensure the condom has been stored in a cool, dry place away from direct sunlight. Do not store condoms in wallet; put the condom on before any genital contact; put the condom on when penis is erect; ensure adequate lubrication during intercourse. If external lubricants are used, use only water-based lubricants. Oil-based or petroleum-based lubricants, such as body lotion or massage oil, can weaken latex condoms. Withdraw while penis is still erect, and hold condom firmly against base of penis. Remove carefully to ensure no semen spills out.

A student nurse asks the instructor about maternal pulse and blood pressure changes during the prenatal period. Which of the following responses from the nurse about cardiovascular changes during the first and second trimesters is accurate? a) "Women experience increased pulse rate and blood pressure." b) "Women experience increased pulse rate and decreased blood pressure." c) "Women experience decreased pulse rate and increased blood pressure." d) "Women experience no change in pulse rate or blood pressure."

b) "Women experience increased pulse rate and decreased blood pressure."

A fetus is assessed at 2 cm above the ischial spines. The nurse would document fetal station as: a) +2 b) -2 c) 0 d) +4

b) -2 Correct Explanation: When the presenting part is above the ischial spines, it is noted as a negative station. Since the measurement is 2 cm, the station would be -2. A 0 station indicates that the fetal presenting part is at the level of the ischial spines. Positive stations indicate that the presenting part is below the level of the ischial spines.

Braxton Hicks contractions are termed "practice contractions" and occur throughout pregnancy. When the woman's body is getting ready to go into labor, it begins to show anticipatory signs of impending labor. Among these signs are Braxton Hicks contractions that are more frequent and stronger in intensity. What differentiates Braxton Hicks contractions from true labor? a) Braxton Hicks contractions cause "ripening" of the cervix. b) Braxton Hicks contractions usually decrease in intensity with walking c) Braxton Hicks contractions get closer together with activity d) Braxton Hicks contractions do not last long enough to be true labor

b) Braxton Hicks contractions usually decrease in intensity with walking Braxton Hicks contractions occur more frequently and are more noticeable as pregnancy approaches term. These irregular, practice contractions usually decrease in intensity with walking and position changes.

A woman calls the prenatal clinic and says that she thinks she might be in labor. She shares her symptoms over the phone with the nurse and asks what to do. The nurse determines that she is likely in true labor and that she should head to the hospital. Which of the following symptoms is an indicator of true labor? a) Increase in fetal kick count b) Contractions beginning in the back and sweeping forward across the abdomen c) Intermittent backache stronger than usual d) Lightening (descent of the fetus into the pelvis)

b) Contractions beginning in the back and sweeping forward across the abdomen True labor contractions usually begin in the back and sweep forward across the abdomen similar to tightening of a rubber band. They gradually increase in frequency and intensity over a period of hours. Lightening and intermittent backache are preliminary signs of labor but do not indicate true labor. Increase in fetal kick count does not indicate true labor.

The nurse is explaining to a primigravida how the zygote becomes implanted into the uterus. How would the nurse describe the structure formed in this process known as the blastocyst? a) A ball of about 16 identical cells is formed when the zygote divides. b) First one, then two layers of cells surround a fluid-filled space. c) The endometrium is enriched in nutrients in preparation for pregnancy. d) A group of cells is forming what will become the embryo.

b) First one, then two layers of cells surround a fluid-filled space. The zygote divides rapidly, until it forms a ball of about 16 identical cells, which is then called a morula. The morula is then swept down the fallopian tube and into the uterus, a process that takes approximately 7 to 9 days. The lining of the uterus, or endometrium, has become rich in nutrients in preparation for the pregnancy. Just before the morula reaches the uterus, the cells begin to form layers; first one, then two layers surround a fluid-filled space, called a blastocyst. Another group of cells form what will become the embryo.

The nurse is teaching a prenatal class the signs and symptoms of true labor. Identify the appropriate response by the class that shows an understanding of true labor contractions. a) Cause discomfort over the top of my uterus. b) Increase even if I relax and take a shower. c) Remain irregular with the same intensity. d) Subside when I walk around and use the lateral position.

b) Increase even if I relax and take a shower. True labor contractions do not stop; they continue and strengthen, as well as increase in frequency. If the contractions subside while taking a shower or relaxing, then they are not labor contractions. The discomfort over the top of the uterus is normal for full term pregnancy.

A pregnant client arrives at the maternity clinic complaining of constipation. Which of the following factors could be the cause of constipation during pregnancy? Select all that apply. a) Reduced stomach acidity b) Increase in estrogen levels c) Intestinal displacement d) Use of iron supplements e) Decreased activity level

b) Increase in estrogen levels c) Intestinal displacement d) Use of iron supplements

When describing the pregnant woman's hypercoagulable state, which of the following would the nurse identify as being least likely related? a) Increased clotting factors b) Increased number of red blood cells c) Increased plasma fibrinogen d) Increased levels of fibrin

b) Increased number of red blood cells

Why is a Papanicolaou smear done at the first prenatal visit? a) It helps to date the pregnancy. b) It identifies abnormal cervical cells. c) It detects if uterine cancer is present. d) It predicts whether cervical cancer will occur.

b) It identifies abnormal cervical cells. A Pap smear is a test for cervical cancer. Should abnormal cells be present, the woman may need to make a decision about her priorities of therapy for cervical disease or continuing the pregnancy.

When describing sexually transmitted infections and testing, the nurse explains that a client is typically tested for chlamydia, gonorrhea, and syphilis at the same time for which reason? a) The infections spread through the same medium, and therefore clients have concurrent infections. b) It is not unusual for clients to have concurrent infections with more than one sexually transmitted infection (STI). c) The infecting bacterium in all cases is the same, and therefore clients have concurrent infections. d) The symptoms of these diseases are the same, and culture tests alone can determine the disease that has infected the client.

b) It is not unusual for clients to have concurrent infections with more than one sexually transmitted infection (STI). It is common practice to test clients for chlamydia, gonorrhea, and syphilis because it is not unusual for clients to have concurrent infections with more than one STI. For chlamydia, the causative microorganism is a bacterium named Chlamydia trachomatis. For gonorrhea, the infection is caused by a bacterium named Neisseria gonorrhoeae. The spirochete Treponema pallidum is the causative microorganism of syphilis. The symptoms of these conditions are not identical. The causative microorganisms do not spread through the same medium

The nurse is examining a woman who came to the clinic because she thinks she is pregnant. Which of the following data collected by the nurse are presumptive signs of her pregnancy? Select all that apply. a) Ultrasound pictures b) Morning sickness c) Amenorrhea d) Fetal heartbeat e) Hydatidiform mole f) Breast changes

b) Morning sickness c) Amenorrhea f) Breast changes

The nurse teaches a pregnant woman about breastfeeding, stating that stimulation of the breast through sucking or touching stimulates the secretion of which hormone? a) Cortisol b) Oxytocin c) Antidiuretic hormone d) Follicle stimulating hormone

b) Oxytocin

When going through the transition phase of labor women often feel out of control. What do women in the transition phase of labor need? a) Just to be left alone b) Positive reinforcement c) Intense nursing care d) Their significant other beside them

b) Positive reinforcement Any women, even ones who have had natural childbirth classes, have a difficult time maintaining positive coping strategies during this phase of labor. Many women describe feeling out of control during this phase of labor. A woman in transition needs support, encouragement, and positive reinforcement.

The placenta is the site where antibodies in the mother's blood pass into the fetal circulation. These antibodies give passive immunity to the fetus for several common childhood diseases. There are some infections for which the mother does not provide antibodies to the fetus. What infection is the fetus not protected from? a) Small pox b) Rubella c) Diptheria d) Rubeola

b) Rubella The fetus does not receive immunity to rubella, cytomegalovirus (CMV), varicella, or measles. If the woman encounters these pathogens during her pregnancy, fetal infection may ensue.

When describing the characteristics of the amniotic fluid to a pregnant woman, which would the nurse include? a) It is composed primarily of organic substances. b) The amount gradually fluctuates during pregnancy. c) It limits fetal movement in utero. d) It is usually an acidic fluid

b) The amount gradually fluctuates during pregnancy. Amniotic fluid is alkaline. Amniotic fluid is composed of 98% water and 2% organic matter. Amniotic fluid volume gradually fluctuates throughout pregnancy. Sufficient amounts promote fetal movement to enhance musculoskeletal development.

A fetus is able to maintain blood circulation in-utero by the presence of circulatory shunts. The ductus arteriosus in-utero shunts blood from: a) The right ventricle to the aorta b) The pulmonary artery to the aorta c) The aorta to the pulmonary veins d) The left to right heart atria

b) The pulmonary artery to the aorta Because the fetal lungs are not inflated, blood must be diverted past them. The ductus arteriosus helps to do this by shunting blood from the pulmonary artery to the aorta. As the heart develops, the septa between the atria and ventricles eventually closes. If it does not, then, depending on the affects, interventions must be planned.

Which of the following statements would the nurse include in the teaching plan for a pregnant woman related to changes in the uterus? a) The uterus reaches its maximum height in the abdomen at 39 weeks. b) The uterus changes from a pear-shaped organ to an oval one. c) Uterine growth occurs because of an increase in the number of cells in the uterus. d) The uterus moves into the abdomen by the second month of pregnancy.

b) The uterus changes from a pear-shaped organ to an oval one.

What should the nurse explain to the pregnant patient about the importance of the fetal stage of development? a) There can be no damage to the fetus if the mother drinks alcohol at this stage. b) There is additional growth and development of the organs and body systems. c) The fetus is ready for delivery. d) Cellular division and implantation occur during this stage of development.

b) There is additional growth and development of the organs and body systems. The fetal stage is from the beginning of the ninth week after fertilization and continues until birth. At this time, the developing human is called a fetus. During the fetal stage, there is additional growth and maturation of the organs and body systems.

A nurse is engaged in primary prevention activities for HPV. The nurse would be most likely involved with which activity? a) encouraging treatment for genital warts b) administering HPV vaccine c) teaching about the importance of regular Pap smears d) educating about HPV testing in women over age 30

b) administering HPV vaccine

What factors would change during a pregnancy if the hormone progesterone were reduced or withdrawn? a. The woman's gums would become red and swollen and would bleed easily. b. The uterus would contract more and peristalsis would increase. c. Morning sickness would increase and would be prolonged. d. The secretion of prolactin by the pituitary gland would be inhibited.

b. The uterus would contract more and peristalsis would increase.

In light of the high incidence of some illnesses in women, which of the following questions is most important to include in a review of systems for a pregnant woman? a) "Do you have a peptic ulcer?" b) "Have you had any neurologic diseases?" c) "Have you had any urinary tract infections?" d) "Have you ever had a heart attack?"

c) "Have you had any urinary tract infections?" Urinary tract infections occur at a greater incidence in pregnant women than in others because stasis of urine occurs because of pressure on the ureters; the trace of glucose often present in urine helps bacteria grow.

During a routine antepartal visit, a pregnant woman reports a white thick vaginal discharge. Which of the following would the nurse do next? a) Check the discharge for evidence of ruptured membranes. b) Tell the woman that this is entirely normal. c) Ask the woman if she is having any itching or irritation. d) Advise the woman about the need to culture the discharge.

c) Ask the woman if she is having any itching or irritation.

Which instruction should the nurse give to a client with genital herpes to help control the infection? a) Apply antibacterial medication. b) Avoid people with upper respiratory infections. c) Avoid sexual contact until sores heal. d) Apply imiquimod cream.

c) Avoid sexual contact until sores heal.

A client is being discharged from the gynecological unit after treatment for an acute pelvic inflammatory disease (PID). What priority instruction regarding disease management should the nurse include? a) Arrange for follow-up visits to her health care provider. b) Refer the client to the women's sexual health clinic. c) Discuss the necessity of completing the antibiotic therapy. d) Provide the client with brochures on sexually transmitted disease prevention.

c) Discuss the necessity of completing the antibiotic therapy.

Which of the following are purposes for prenatal care? (Select all that apply.) a) Increase the business of the clinic. b) Maximize the risk of possible complications. c) Establish a baseline of present health. d) Determine the gestational age of the fetus. e) Identify women at risk for complications. f) Monitor for fetal development and maternal well-being.

c) Establish a baseline of present health. d) Determine the gestational age of the fetus. e) Identify women at risk for complications. f) Monitor for fetal development and maternal well-being. The purposes of prenatal care are to establish a baseline of present health; determine the gestational age of the fetus; monitor fetal development and maternal well-being; identify women at risk for complications and minimize the risk of possible complications; and provide time for education about pregnancy, lactation, and newborn care. It is not done to help a clinic financially.

A client in her second trimester of pregnancy is anxious about the blotchy, brown pigmentation appearing on her forehead and cheeks. She also complains of increased pigmentation on her breasts and genitalia. When educating the client, which of the following would the nurse identify as the condition experienced by the client? a) Vascular spiders b) Linea nigra c) Facial melasma (cholasma) d) Striae gravidarum

c) Facial melasma (cholasma)

The RN in labor and delivery documents the fetus as ROA. To what does this documentation refer for a fetus? a) Fetal size b) Fetal station c) Fetal position d) Fetal attitude

c) Fetal position When documenting the ROA, this is the Right Occiput Anterior or the relationship of the fetal position to the mother using the maternal pelvis as the point of reference. Fetal refers to the relationship of the presenting part of the fetus to the ischial spines of the pelvis. Fetal attitude refers to the relationship of the fetal parts to one another. Fetal size refers the actual size of the developing fetus.

A client in her 39th week of gestation arrives at the maternity clinic stating that earlier in her pregnancy, she experienced shortness of breath. However, for the past few days, she's been able to breathe easily, but she has also begun to experience increased urinary frequency. A nurse is assigned to perform the physical examination of the client. Which of the following is the nurse most likely to observe? a) Fundal height is at its highest level at the xiphoid process. b) The lower uterine segment and cervix have softene c) Fundal height has dropped since the last recording. d) The fundus is at the level of the umbilicus and measures 20 cm.

c) Fundal height has dropped since the last recording.

A pregnant woman you care for tells you she often has allergic responses to drugs. She is concerned that she will be allergic to her fetus or her body will reject the pregnancy. You would base your reply to her on which of the following statements? a) The kidneys release a hormone during pregnancy to prevent this from happening. b) The level of aldosterone during pregnancy reduces production of IgG antibodies. c) Immunologic activity is decreased during pregnancy. d) The decreased corticosteroid activity during pregnancy ensures this will not happen.

c) Immunologic activity is decreased during pregnancy.

A 28-year-old primigravida client with diabetes mellitus, in her first trimester, comes to the health care clinic for a routine visit. The client reports frequent episodes of sweating, giddiness, and confusion. What should the nurse tell the client about these experiences? a) Insulin resistance becomes minimal in the latter half of the pregnancy b) Tissue sensitivity to insulin increases as pregnancy advances c) Increased secretion of insulin occurs in the first trimester d) Use of insulin needs to be reduced as pregnancy advances

c) Increased secretion of insulin occurs in the first trimester

During a routine antepartal visit, a pregnant woman says, "I've noticed my gums bleeding a bit since I've become pregnant. Is this normal?" The nurse bases the response on the understanding of which of the following? a) Increased venous pressure leads to increased gingival friability. b) Elevated progesterone levels cause smooth muscle relaxation. c) Influence of estrogen and blood vessel proliferation d) Effects of regurgitation from relaxation of the cardiac sphincter

c) Influence of estrogen and blood vessel proliferation

The nurse is preparing a teaching plan for a pregnant woman about the signs and symptoms to be reported immediately to her health care provider. Which of the following would the nurse include? Select all that apply. a) Backache during the second trimester b) Nausea with vomiting during the first trimester c) Lower abdominal pain with shoulder pain in the first trimester d) Sudden leakage of fluid during the second trimester e) Headache with visual changes in the third trimester f) Urinary frequency in the third trimester

c) Lower abdominal pain with shoulder pain in the first trimester d) Sudden leakage of fluid during the second trimester e) Headache with visual changes in the third trimester Danger signs and symptoms that need to be reported immediately include headache with visual changes and sudden leakage of fluid in the second trimester, and lower abdominal pain accompanied by shoulder pain in the first trimester. Urinary frequency in the third trimester, nausea and vomiting during the first trimester, and backache during the second trimester are common discomforts of pregnancy.

A woman in the third trimester of her first pregnancy expresses fear about the birth canal being wide enough for her to push the baby through it during labor. She is a petite person, and the baby seems so large. She asks the nurse how this will be possible. To help alleviate the patient's fears, the nurse should mention the role of the hormone that softens the cervix and collagen in the joints, which allows dilation and enlargement of the birth canal. This hormone is which of the following? a) Estrogen b) Human placental lactogen c) Relaxin d) Progesterone

c) Relaxin

The fetus receives blood flow from the mother via the placenta and umbilical cord. What is the route of fetal circulation through the umbilical cord? a) The two umbilical veins carry waste products from the fetus to the placenta b) The two umbilical arteries carry waste products from the placenta to the fetus c) The one umbilical vein carries oxygen rich blood to the fetus from the placenta d) The one umbilical artery carries oxygen rich blood to the fetus from the placenta

c) The one umbilical vein carries oxygen rich blood to the fetus from the placenta There are two umbilical arteries and one umbilical vein. The arteries carry waste from the fetus to the placenta; the vein carries oxygenated blood to the fetus from the placenta.

A patient who has just given a blood sample for pregnancy testing in the doctor's office asks the nurse what method of confirming pregnancy is the most accurate. The nurse explains the difference between presumptive symptoms, probable signs, and positive signs. Which of the following should the nurse mention as an example of a positive sign, which may be used to diagnose pregnancy? a) Laboratory test of a urine specimen for hCG b) Laboratory test of a blood serum specimen for hCG c) Visualization of the fetus by ultrasound d) Absence of a period

c) Visualization of the fetus by ultrasound

Clients who have had PID are prone to which complication? a) multiple gestation b) ovarian cancer c) ectopic pregnancy d) inguinal lymphadenopathy

c) ectopic pregnancy

Reva Rubin identified four major tasks that the pregnant woman undertakes to form a mutually gratifying relationship with her infant. What is "binding in"? a. Ensuring safe passage through pregnancy, labor, and birth b. Seeking acceptance of this infant by others c. Seeking acceptance of self as mother to the infant d. Learning to give of oneself on behalf of the infant

c. Seeking acceptance of self as mother to the infant

The patient is having a routine prenatal visit and asks the nurse what the childbirth education teacher meant when she used the term zero station. What is the best response by the nurse? a) This indicates that you start labor within the next 24 hours." b) "This is just a way of determining your progress in labor." c) "This means +1 and the baby is entering the true pelvis." d) "The presenting part is at the true pelvis and is engaged."

d) "The presenting part is at the true pelvis and is engaged." Zero station is when the fetus is engaged in the pelvis, or has dropped. This is an encouraging sign for the patient. This sign is indicative that labor may be beginning, but there is no set time frame regarding when it will start. Labor has not started yet, and the fetus has not begun to move out of the uterus.

A woman in the second trimester of pregnancy reports that she is "tired all the time." She appears pale and her hematocrit, though within the normal range, is low. Which recommendation would be most helpful for this woman? a) A calcium supplement b) More seafood and organ meats in her diet c) More meat in her diet d) An iron supplement

d) An iron supplement

As a pregnant woman lies on the examining table, she grows very short of breath and dizzy. This phenomenon probably happens because a) Her cerebral arteries are growing congested with blood b) Sympathetic nerve responses cause dyspnea when a woman lies supine c) The uterus requires more blood in a supine position d) Blood is trapped in the vena cava in a supine position

d) Blood is trapped in the vena cava in a supine position

What are the two fetal membranes? a) Amnion and mesoderm b) Ectoderm and amnion c) Amnion and endoderm d) Chorion and amnion

d) Chorion and amnion The chorion and amnion are the two fetal membranes. The ectoderm, mesoderm, and endoderm are layers in the developing blastocyst.

A 40-year-old patient is in her 16th week of pregnancy. So far, her pregnancy appears to be healthy, with no abnormal results from standard diagnostic tests. Because of her age, however, the patient is a candidate for which diagnostic tests? (Select all that apply.) a) Nuchal translucency screening b) Amniocentesis c) Maternal serum screening d) Chorionic villi sampling

d) Chorionic villi sampling b) Amniocentesis Chorionic villi sampling (CVS) and amniocentesis are both techniques that may be offered to women who are older than 35 years of age, or to those whose MSAFP level is abnormal, to further screen for genetic disorders. Women of all ages are offered routine sonogram screening (a nuchal translucency scan) and analysis of maternal serum levels of alpha-fetoprotein (MSAFP) by a quadruple screen early in pregnancy to evaluate for neural tube, abdominal wall, or chromosomal disorders in the fetus.

During pregnancy a woman has many psychological adaptations that must be made. The nurse must remember that the baby's father is also experiencing the pregnancy and has adaptations that must be made. Some fathers actually have symptoms of the pregnancy along with the mothers. What is this called? a) Pseudo pregnancy b) Cretinism c) Pregnancy syndrome d) Couvade syndrome

d) Couvade syndrome

The nurse notes that the fetal head is at the vaginal opening and does not regress between contractions. The nurse interprets this finding as which of the following? a) Descent b) Engagement c) Restitution d) Crowning

d) Crowning Crowning occurs when the top of the fetal head appears at the vaginal orifice and no longer regresses between contractions. Engagement occurs when the greatest transverse diameter of the head passes through the pelvic inlet. Descent is the downward movement of the fetal head until it is within the pelvic inlet. Restitution or external rotation occurs after the head is born and free of resistance. It untwists, causing the occiput to move about 45 degrees back to its original left or right position.

If a woman is 3 months pregnant, which of the following findings related to breast changes would you expect to assess? a) Slack, soft breast tissue b) Enlarged lymph nodes c) Deeply fissured nipples d) Darkened breast areolae

d) Darkened breast areolae

Which of the following would the nurse identify as a normal physiologic change in the renal system due to pregnancy? a) Shortening of the ureters b) Decrease in glomerular filtration rate c) Reduction in kidney size d) Dilation of the renal pelvis

d) Dilation of the renal pelvis

When teaching a group of nursing students about the stages of labor, the nurse explains that softening, thinning, and shortening of the cervical canal occur during the first stage of labor. Which of the following terms is the nurse referring to in the explanation? a) Crowning b) Molding c) Dilatation d) Effacement

d) Effacement The nurse is explaining about effacement, which involves softening, thinning, and shortening of the cervical canal. Dilatation refers to widening of the cervical os from a few millimeters in size to approximately 10 cm wide. Crowning refers to a point in the maternal vagina from where the fetal head cannot recede back after the contractions have passed. Molding is a process in which there is overriding and movement of the bones of the cranial vault, so as to adapt to the maternal pelvis.

Which of the following is a positive sign of pregnancy? a) Positive pregnancy test b) Hegar's sign c) Uterine contractions d) Fetal movement felt by examiner

d) Fetal movement felt by examiner

A woman in her 15th week of pregnancy is about to undergo amniocentesis. Which of the following nursing interventions should be made first? a) Have the patient void b) Place the patient in supine position c) Observe the fetal heart rate monitor d) Obtain a signed consent form

d) Obtain a signed consent form Nursing responsibilities for assessment procedures include seeing a signed consent form has been obtained as needed (necessary if the procedure poses any risk to the mother or fetus that would not otherwise be present, as is the case with amniocentesis). All of the answers are nursing interventions that should be made before or during amniocentesis, but having the patient sign a consent form should be completed before the others.

A nurse is performing a vaginal examination of a woman in the early stages of labor. The woman has been at 2 cm dilated for the past 2 hours, but effacement has progressed steadily. Which of the following should the nurse do to best encourage the client regarding her progress? a) Say, "there has been no further dilatation; effacement is progressing." b) Say, "you haven't dilated any further, but hang in there; it will happen eventually." c) Don't mention anything to the client yet; wait for further dilatation to occur. d) Say, "you are still 2 cm dilated, but the cervix is thinning out nicely."

d) Say, "you are still 2 cm dilated, but the cervix is thinning out nicely." Women are anxious to have frequent reports during labor, to reassure them everything is progressing well. If giving a progress report, remember most women are aware of the word dilatation but not effacement. Just saying, "no further dilatation", therefore, is a depressing report. "You're not dilated a lot more, but a lot of thinning is happening and that's just as important" is the same report given in a positive manner.

Implantation generally occurs at which place on the uterus? a) Directly over the cervical os b) Directly over an opening to a fallopian tube c) The lower anterior surface d) The upper posterior surface

d) The upper posterior surface Implantation occurs most commonly on the upper posterior surface of the uterus. This position allows the fetus to deliver before the placenta.

What should the nurse explain to the pregnant patient about the importance of the fetal stage of development? a) Cellular division and implantation occur during this stage of development. b) The fetus is ready for delivery. c) There can be no damage to the fetus if the mother drinks alcohol at this stage. d) There is additional growth and development of the organs and body systems.

d) There is additional growth and development of the organs and body systems. The fetal stage is from the beginning of the ninth week after fertilization and continues until birth. At this time, the developing human is called a fetus. During the fetal stage, there is additional growth and maturation of the organs and body systems.

What should the nurse explain to the pregnant patient about the importance of the fetal stage of development? a) Cellular division and implantation occur during this stage of development. b) The fetus is ready for delivery. c) There can be no damage to the fetus if the mother drinks alcohol at this stage. d) There is additional growth and development of the organs and body

d) There is additional growth and development of the organs and body systems. The fetal stage is from the beginning of the ninth week after fertilization and continues until birth. At this time, the developing human is called a fetus. During the fetal stage, there is additional growth and maturation of the organs and body systems.

A female client with genital herpes is prescribed acyclovir as treatment. After teaching the client about this treatment, which statement by the client indicates effective teaching? a) "This drug will help reduce my risk for a recurrence after discontinuing it." b) "The severity of future attacks will be much less after using this drug." c) "If I use this drug, I will be cured of the infection." d) "This drug will help to suppress any symptoms of the infection."

d) This drug will help to suppress any symptoms of the infection. No cure exists, but antiviral drug therapy helps to reduce or suppress symptoms, shedding, and recurrent episodes. Advances in treatment with acyclovir 400 mg orally three times daily for 7 to 10 days, famciclovir 250 mg orally three times daily for 7 to 10 days, or valacyclovir 1 g orally twice daily for 7 to 10 days have resulted in an improved quality of life for those infected with HSV. However, according to the CDC, these drugs neither eradicate latent virus nor affect the risk, frequency, or severity of recurrences after the drug is discontinued.

A client has been admitted with primary syphilis. Which signs or symptoms should the nurse expect to see with this diagnosis? a) copper-colored macules on the palms and soles that appeared after a brief fever b) patchy hair loss and red, broken skin involving the scalp, eyebrows, and beard areas c) one or more flat, wartlike papules in the genital area that are sensitive to touch d) a painless genital ulcer that appeared about 3 weeks after unprotected sex

d) a painless genital ulcer that appeared about 3 weeks after unprotected sex. A painless genital ulcer is a symptom of primary syphilis. Macules on the palms and soles after fever are indicative of secondary syphilis, as is patchy hair loss. Wartlike papules are indicative of genital warts.

Which finding would the nurse most likely find in a male diagnosed with a chlamydia trachomatis infection? a) hematuria b) painful ejaculation c) erectile dysfunction d) dysuria

d) dysuria

An 18-year-old pregnant woman asks why she has to have a routine alpha-fetoprotein serum level drawn. You explain that this a) tests the ability of her heart to accommodate the pregnancy. b) measures the fetal liver function. c) is a screening test for placental function. d) may reveal chromosomal abnormalities.

d) may reveal chromosomal abnormalities. An alpha-fetoprotein analysis is a cost-effective screening test to detect chromosomal and open-body-cavity disorders.

A feeling expressed by most women upon learning they are pregnant is: a. Acceptance b. Depression c. Jealousy d. Ambivalence

d. Ambivalence

When educating a group of nursing students about the theories of onset of labor, the nurse identifies which of the following factors as the possible causes for onset of labor? Select all that apply. a) Prostaglandin production in the myometrium b) Release of oxytocin by the pituitary c) Fall in the estrogen at 34-35 weeks of pregnancy d) Increase in the production of progesterone e) Increase in the fetal cortisol levels

e) Increase in the fetal cortisol levels b) Release of oxytocin by the pituitary c) Prostaglandin production in the myometrium Explanation: The possible causes for the onset of labor include increase in the fetal cortisol levels, release of oxytocin by the posterior pituitary and the production of prostaglandins. Progesterone withdrawal, and not an increase, initiates labor. There is a rise in the estrogen levels at 34-35 weeks of pregnancy. Estrogen stimulates prostaglandin production and also promotes the release of oxytocin.

After teaching a pregnant client who is in her last weeks of pregnancy about the signs and symptoms of approaching labor, which of following if identified by the client would indicate effective teaching? Select all that apply. a) Weight gain b) Backache c) Constipation d) Bloody show e) Lightening

e) Lightening d) Bloody show b) Backache The signs of approaching labor include lightening, bloody show, and backache. Lightening is the falling forward of the pregnant uterus due to settlement of the fetal head into the maternal pelvis. Backache associated with pelvic cramping pain, which is regular and increases in the intensity, is suggestive of impending labor. Bloody show is the expulsion of the cervical mucus plug tinged with blood, and occurs due to cervical effacement and dilatation. Weight loss and diarrhea are other signs of impending labor. Weight gain and constipation are not signs of impending labor.

The nurse is creating an educational pamphlet for pregnant mothers. Which is the best description of fetal development for the nurse to emphasize?

gestational age, length, weight, and systems developed Client education is a major component of maternal-child nursing. During pregnancy, nurses provide anticipatory guidance to prepare the woman and her significant other for the changes each month brings. Clients most often want to know gestational age in weeks, length, weight, and systems developed; the client is then able to visualize what the fetus looks like.

A primary care provider tells a client to return 2 to 3 months after treatment to have a repeat culture done to verify the cure. This prescription would be appropriate for a woman with which condition?

gonorrhea

A nurse is conducting an assessment of a client who is suspected of having pelvic inflammatory disease. Which finding would the nurse identify as mandatory for the diagnosis to be made? Select all that apply.

lower abdominal tenderness adnexal tenderness cervical motion tenderness To reduce the risk of missed diagnosis, the CDC has established criteria for the diagnosis of PID. Minimal criteria (all must be present) are lower abdominal tenderness, adnexal tenderness, and cervical motion tenderness. Additional supportive criteria that support a diagnosis of PID include abnormal cervical or vaginal mucopurulent discharge and oral temperature above 101° F (38.3° C)

The purpose of a circulatory shunt, such as the ductus arteriosus, is to a) allow fetal blood and maternal blood to mix freely. b) direct blood flow to the lungs to supply nutrients necessary for growth and maturation. c) allow blood to bypass the fetal heart, which does not function as a pump until birth. d) permit oxygenated blood to supply the most important fetal organs.

permit oxygenated blood to supply the most important fetal organs. The ductus arteriosus helps propel oxygenated blood quickly to the coronary arteries, brain, and kidneys.

A client who has been treated for recurring gonorrhea visits the sexual health clinic reporting dysuria, urinary frequency, and whitish-yellow vaginal discharge. A pelvic examination reveals cervicitis. How would the nurse interpret these findings?

the possibility of a chlamydia infection Any woman suspected of having gonorrhea should be tested for chlamydia also because coinfection (45%) is extremely common. The nursing management of chlamydia and gonorrhea includes treatment of both. The prevalence of chlamydia and gonorrhea is increasing dramatically.


Conjuntos de estudio relacionados

Virginia Life: Life Insurance Policies (A-B)

View Set

Terms, History of Photo Final Exam

View Set

Unit 2 Checkpoint 3 Open Notes Quiz

View Set

Psychology and the Legal System Ch 1, Psychology and the Legal System Ch2, Psychology and the Legal System Ch3 Part 1

View Set

Finance 300 Study Vocabs Exam 2 Fall 2020

View Set